Está en la página 1de 98

Índice general

1 Preliminares 3
1.1 Lógica . . . . . . . . . . . . . . . . . . . . . . . . . . . . . . . . . . . . 3
1.2 Conjuntos . . . . . . . . . . . . . . . . . . . . . . . . . . . . . . . . . . 5
1.3 Soluciones a Algunos Ejercicios . . . . . . . . . . . . . . . . . . . . . . 11

2 El Principio de Inducción 13
2.1 Los Números Naturales . . . . . . . . . . . . . . . . . . . . . . . . . . 13
2.2 Variantes de Inducción . . . . . . . . . . . . . . . . . . . . . . . . . . 15
2.3 Ejercicios de Inducción . . . . . . . . . . . . . . . . . . . . . . . . . . 17

3 Espacios Vectoriales 23

4 Subespacios 31
4.1 Subespacio vectorial . . . . . . . . . . . . . . . . . . . . . . . . . . . . 31

5 Bases, dimensión, Fórmula de Grassmann 37


5.1 Problemas . . . . . . . . . . . . . . . . . . . . . . . . . . . . . . . . . 44

6 Coordenadas, cambio de coordenadas 49


6.1 Ejercicios . . . . . . . . . . . . . . . . . . . . . . . . . . . . . . . . . . 55

7 Aplicaciones lineales 59
7.1 Problemas . . . . . . . . . . . . . . . . . . . . . . . . . . . . . . . . . 67

8 Teorı́a del Endomorfismo 69


8.1 El polinomio mı́nimo . . . . . . . . . . . . . . . . . . . . . . . . . . . 79
8.2 Problemas . . . . . . . . . . . . . . . . . . . . . . . . . . . . . . . . . 81
8.3 PageRank . . . . . . . . . . . . . . . . . . . . . . . . . . . . . . . . . . 82
8.4 Números enteros . . . . . . . . . . . . . . . . . . . . . . . . . . . . . . 88

1
1 Preliminares
1.1. Lógica
En esta sección damos un breve repaso a la lógica y al lenguaje que vamos a utilizar a
lo largo del curso.
Intuitivamente, una proposición es una afirmación al que le podemos dar un valor de
verdad, verdadero o falso.

1.1.1. Tablas de verdad de AND, OR, NOT, →


Definición 1.1.1. Dadas dos proposiciones p y q que pueden interpretarse como verda-
deras o falsas, podemos construir nuevas proposiciones.
• p ∧ q, p y q es la proposición que es verdadera exactamente en el caso de que ambas
proposiciones p, q son verdaderas.
• p ∨ q, p o q. Esta proposición es verdadera cuando al menos una de las dos
proposiciones p, q es verdadera. Nótese que esta disyunción es inclusiva, si ambas p
y q son verdaderas, p ∨ q también será verdadera.
• ¬p, no p, la negación de la proposición p.
• p → q, p implica q, si p entonces q, Esta proposición lo que quiere decir es lo
siguiente: si p es cierta, debe ser q también cierta para que p → q sea cierto. Si p es
falsa, p → q no afirma nada sobre la veracidad de q. Si p es falsa, p → q siempre es
cierto.
Dependiendo del valor de veracidad de p y q, podemos describir el valor de verdad de
p ∧ q, p ∨ q, negp, p → q mediante una tabla.
p q p AND q p q p OR q
V V V V V V
V F F V F V
F V F F V V
F F F F F F

p q p→q
V V V p NOT p
V F F V F
F V V F V
F F V

3
1 Preliminares

Recuerda que:

• NOT (p AND q) ≃ (NOT p) OR (NOT q)

• NOT (p OR q) ≃ (NOT p) AND (NOT q)

• p → q ≃ (NOT p) OR q

• NOT (p → q) ≃ p AND (NOT q)

Ejercicio 1.1.2. Describe ejemplos que muestren las anteriores equivalencias

1.1.2. Cuantificadores
Dada una propiedad P (x) que depende de una variable, por ejemplo x es un alumno de
Álgebra Lineal y Discreta, podemos crear predicados cuantificando sobre dicha propiedad.
Los cuantificadores que usaremos son “para todo” (sı́mbolo ∀) y “existe” (sı́mbolo ∃).

• ∀x ∈ A(p(x)). Significa que todos y cada uno de los elementos de A cumplen la


propiedad p. Para todo x en A, p.

• ∃x ∈ A(p(x)). Significa que existe al menos un elemento de A que cumple la


propiedad p. Existe un x en A tal que p.

Claramente, si una propiedad p se cumple para todo elemento de A (y A no es vacı́o)


entonces existe un elemento en A que cumple p.

Ejercicio 1.1.3. Escribe la negación de las siguientes afirmaciones.

1. Todos los hombres son mortales.

2. Todos los fantasmas son blancos.

3. Existe un marciano.

4. Para toda persona p existe una mujer m tal que m es la madre de p (todas las
personas tienen madre).

5. Si hoy llueve me mojo.

6. Pienso luego existo.

Ejercicio 1.1.4. ¿Son las siguientes sentencias equivalentes?

• Para toda persona p existe una mujer m tal que m es la madre de p.

• Existe una mujer m tal que para toda persona p, m es la madre de p.

Los cuantificadores no se pueden intercambiar sin cambiar el significado.

4
1.2 Conjuntos

1.2. Conjuntos
Admitiremos una idea intuitiva de conjunto como una colección o familia de objetos
relacionados por alguna propiedad común. Un conjunto queda completamente determinado
por sus elementos, es decir, dos conjuntos son iguales si y sólo si tienen exactamente los
mismos elementos.
La relación básica de los conjuntos es la relación de pertenencia, que se denota
por ∈. Si B es un conjunto y x es un elemento de B escribiremos x ∈ B, que se lee x
pertenece a B. Por otro lado, si x no pertenece a B, escribiremos x ∈ / B.
Una forma de describir conjuntos pequeños es por extensión, listando sus elementos
entre llaves

Ejemplo 1.2.1. En este ejemplo, A es el conjunto formado por el 1, el 2 y el 3.

A = {1, 2, 3}

1∈A
8∈
/A
Nótese que un conjunto puede ser a su vez un elemento de otro conjunto.

B = {{1, 2, 3}, {a}, {a, b}, 4}


4∈B
a∈
/B
{a} ∈ B
B tiene cuatro elementos, tres de ellos son a su vez conjuntos.

• Dado un conjunto A y un elemento x, debemos saber (al menos en principio) sin


ambigüedad si x es un elemento de A.

Axioma Un conjunto está completamente determinado por sus elementos. Es decir,


dos conjuntos A y B son el mismo si y solo si tienen los mismos elementos.

Ejemplo 1.2.2.

• A = {1, 3}, B = {0 + 1, 4 − 1}. En este caso A = B son conjuntos iguales, el conjunto


A consta de los números 1 y 3. El conjunto B también está formado por los números
1 y 3, pero están representados de otra manera.

• A = {1, 3}, B = {1, 3, 4}. En este caso A ̸= B son conjuntos distintos, 4 pertenece a
B pero no a A.

• A = {1, 1, 2}, B = {2, 2, 1, 1}, A y B son iguales, pues ambos tienen los mismos
elementos, el 1 y el 2. No importa el orden en el que estén los elementos y no hay
repeticiones.

5
1 Preliminares

Definición 1.2.3. Sean A y B conjuntos, diremos que A está contenido en B y lo


denotaremos por A ⊆ B si todo elemento de A es elemento de B. A ⊆ B es una
abreviatura de
x∈A→x∈B

Ejemplo 1.2.4.

• Sea A = {1, 2, 3}, B = {1, 2, 3, 4}. Claramente A ⊆ B, pues todos los elementos de
A son elementos de B. Sin embargo B ̸⊆ A pues 4 ∈ B pero no es un elemento de A.

Observación 1.2.5. Para comprobar A ⊆ B debemos demostrar que todos y cada uno de
los elementos de A son elementos de B. Sin embargo, para comprobar A ̸⊆ B debemos
encontrar un elemento que está en A pero no esté en B.

Teorema 1.2.6. Si A es cualquier conjunto, entonces A ⊆ A

Demostración. Todo elemento de A es un elemento de A.

Teorema 1.2.7. Si A y B son conjuntos entonces A = B si y solo si A ⊆ B y B ⊆ A.

Demostración. El teorema nos dice dos cosas, que si A = B entonces A ⊆ B y B ⊆ A.


Esto está claro por el Teorema 1. Por otro lado, este teorema afirma que si A ⊆ B y
B ⊆ A, entonces A = B. Esta afirmación también se cumple, pues si x es un elemento de
A, como A ⊆ B entonces x ∈ B. De la misma manera, si x ∈ B entonces como A ⊆ B,
tendremos también que B ⊆ A. Observamos que A y B tienen exactamente los mismos
elementos y por tanto A = B.

Hay varios conjuntos que tienen una notación definida por la tradición

• N el conjunto de los números naturales.

• Z el conjunto de los números enteros.

• Q el conjunto de los números racionales.

• R el conjunto de los números reales.

• C el conjunto de los números complejos.

La forma más común de describir los conjuntos es mediante una propiedad que distinga
los elementos del conjunto de los que no están

A = {x ∈ N | 0 ≤ x ≤ 100}
A es el conjunto de todos los números naturales entre 0 y 100 (inclusive). La barra
vertical se lee “tal que”. Esta manera de describir un conjunto suele tomar la estructura:

A = {x ∈ B | x cumple P }
A es el conjunto formado por todos los elementos de B que verifican la propiedad P .

6
1.2 Conjuntos

¿Cuál serı́a el conjunto?

B = {x ∈ R | x2 + 1 = 0}

B es el conjunto de los números reales x que son solución de la ecuación x2 + 1 = 0.


Esta ecuación no tiene soluciones reales, por lo que el conjunto no tiene elementos. Nótese
que los elementos de B están descritos sin ambigüedad. Si x es un objeto, x no está en B
pues o bien no es un número real o, si es un número real, x2 + 1 ̸= 0. Tenemos que B
es un conjunto sin elementos. Este conjunto especial se llama el conjunto vacı́o y se
denota por ∅.
Propiedades del conjunto vacı́o:
Sea A cualquier conjunto y x un objeto cualquiera, entonces:

• ∅ ⊆ A.

• x∈
/ ∅.

En particular:

• ∅ ⊆ ∅.

• ∅∈
/ ∅.

Efectivamente, ∅ ⊆ A. Pues si x es un elemento de ∅ entonces x es un elemento de A.


Como nunca se podrá cumplir la premisa, la implicación es cierta. Para verlo de otra
manera veamos que la negación ∅ ̸⊆ A no se cumple. Para que se cumpla ∅ ̸⊆ ∅ debe
haber un elemento de ∅ que no esté en ∅. Eso es imposible, por lo que ∅ ̸⊆ ∅ no se cumple
y es ∅ ⊆ ∅ lo que se cumple.

1.2.1. Operaciones Básicas de Conjuntos


En contraposición al conjunto vacı́o muchas veces consideraremos también un conjunto
universal, que denotaremos por E. En presencia de un conjunto universal supondremos
que los elementos con los que vamos a manejar son los elementos de E.
Las operaciones básicas con conjuntos son:

Definición 1.2.8. Dados dos conjuntos A y B, la unión de A y B es el conjunto cuyos


elementos son los elementos que están en A o en B. Se denota por A ∪ B.

Definición 1.2.9. Dados dos conjuntos A y B, la intersección de A y B es el conjunto


cuyos elementos son los elementos que están en A y en B. Se denota por A ∩ B. Dos
conjuntos A y B tales que A ∩ B = ∅ se denominan disjuntos.

Definición 1.2.10. Dado un conjunto A ⊆ E con elementos en un conjunto universal,


el complementario de A es el conjunto cuyos elementos son aquellos de E que no están
en A. Lo denotaremos por A′ . Tendremos que x ∈ A′ si y sólo si x ∈ A.

7
1 Preliminares

Definición 1.2.11. Dados dos conjuntos A y B, definimos la diferencia de A y B y


denotamos por A − B al conjunto cuyos elementos son aquellos elementos que están en
A pero no están en B.

Ejercicio 1.2.12. Sean A, B, C conjuntos:

1. A ∪ A = A ∩ A = A.

2. A ∪ ∅ = A.

3. A ∩ ∅ = ∅.

4. A ∪ B = B ∪ A.

5. A ∩ B = B ∩ A.

6. (A ∪ B) ∪ C = A ∪ (B ∪ C).

7. (A ∩ B) ∩ C = A ∩ (B ∩ C).

8. (A ∪ B) ∩ C = (A ∩ C) ∪ (B ∩ C).

9. (A ∩ B) ∪ C = (A ∪ C) ∩ (B ∪ C).

Ejercicio 1.2.13. Consideremos un conjunto E = {1, 2, 3, 4, 5, 6, 7, 8, 9} y tomemos los


subconjuntos A = {1, 2, 3}, B = {1, 3}, C = {2, 5, 7}. Calcula los siguientes conjuntos:

1. A ∪ B ∪ C

2. A ∩ B ∩ C

3. A\B, B\A, A\C, C\A, B\C, C\B

Comprueba que en este caso concreto se verifican las siguientes igualdades:

1. (A ∪ B) ∩ C = (A ∩ C) ∪ (B ∩ C)

2. (A ∪ B) ∩ C = (A ∩ C) ¿se cumplirá esta igualdad para conjuntos arbitrarios?

3. A ∪ (B ∩ C) = (A ∪ B) ∩ (A ∪ C)

4. (A ∪ B)′ = A′ ∩ B ′

5. (A ∩ B)′ = A′ ∪ B ′

Ejercicio 1.2.14. Muestra dos conjuntos A, B tales que A\B = A, B\A = B. Para ese
conjunto A calcula un conjunto C tal que A\C = ∅ y C\A = ∅ ¿qué relación deben tener
A y C en este caso?

Ejercicio 1.2.15. Sea A un conjunto cualquiera. Calcula A ∩ ∅ y A ∪ ∅.

Ejercicio 1.2.16. Demuestra:

8
1.2 Conjuntos

1. A ⊆ A ∪ B

2. A ∩ B ⊆ B

3. A′′ = A

4. Leyes de Morgan:
a) (A ∪ B)′ = A′ ∩ B ′
b) (A ∩ B)′ = A′ ∪ B ′

5. A\B = A\(A ∩ B)

6. A ⊆ A

7. A′ = E\A

8. E ′ = ∅

9. ∅′ = E

10. Si A ⊆ B entonces B ′ ⊆ A′

1.2.2. Ejercicios Adicionales


Los ejercicios siguientes se han extraı́do de la lectura recomendada:
Matemáticas I, Ingeniero en Informática. Autor: J. M. de Olazábal
Parte de ellos se han tomado de la asignatura Matemàtiques I de la facultad de
informática de la Universidad Politécnica de Cataluña.
Ejercicio 1.2.17. Enumera los elementos de los conjuntos siguientes:
1. {x | x ∈ R, x2 = 1}

2. {x | x ∈ Z, 0 < x < 12}

3. {x ∈ Z | x2 = 2}

4. {x ∈ Z | x < 100, x2 ∈ Z}
Ejercicio 1.2.18. Da una propiedad para describir los conjuntos siguientes:
1. {0, 3, 6, 9, 12, 15}

2. {−3, −2, −1, 0, 1, 2, 3}

3. {−2, 2}

4. {a, e, i, o, u}
Ejercicio 1.2.19. Sea A = {2, 4, 6}, B = {2, 6}, C = {4, 6}, D = {4, 6, 8}
¿Cuáles son subconjuntos de otro distinto?

9
1 Preliminares

Ejercicio 1.2.20. Di a qué conjuntos de los siguientes pertenece el 2


1. {x ∈ R | 1 < x, x ∈ Z}

2. {x ∈ R | ∃y ∈ Z, x = ± y}

3. {2, {2}}

4. {{2}, {2, {2}}}


Nota: Asumimos que A ∈ / A sea cual sea el conjunto A. Es decir conjuntos del tipo
x = {x} están prohibidos.
Ejercicio 1.2.21. Determina si las siguientes proposiciones son ciertas o falsas

x ∈ {x} {x} ⊆ {x} {x} ∈ {x} {x} ∈ {{x}} ∅ ⊆ {x} ∅ ∈ {x}

Ejercicio 1.2.22. ¿Cuántos elementos tienen los siguientes conjuntos?

{a} {{a}} {a, {a}}

Ejercicio 1.2.23. ¿Es cierto que P (A) = P (B) → A = B?


Ejercicio 1.2.24. ¿Cuántos elementos tienen los siguientes conjuntos?

P ({a, b, {a, b}}) P ({∅, a, {a}, {{a}}})

Ejercicio 1.2.25. Sea A el conjunto de los estudiantes que viven a menos de 2 kilómetros
de la Facultad y B el conjunto de los estudiantes que van a pie a clase. Describir:

A∪B A∩B A\B B\A

¿A cuales de los 4 conjuntos anteriores perteneces?


Ejercicio 1.2.26. Sea A el conjunto de habitantes de Santander y B el conjunto de
estudiantes que cursan 1º de informática. Describir en términos de A y B cada uno de
los siguientes conjuntos:
1. El conjunto de los habitantes de Santander que estudian informática

2. El conjunto de habitantes de Santander que no estudian informática

3. El conjunto de los españoles que residen en Santander o estudian informática

4. El conjunto de los españoles que ni residen en Santander ni estudian informática.


Ejercicio 1.2.27. Sean A = {1, 2, 3, 4, 5} y B = {0, 3, 6} Describe:

A∪B A∩B A\B B\A

Ejercicio 1.2.28. Encuentra dos conjuntos A y B tales que:

A ∩ B = {3, 6, 9} A\B = {1, 5, 7, 8} B\A = {2, 10}

10
1.3 Soluciones a Algunos Ejercicios

1.3. Soluciones a Algunos Ejercicios


Ejercicio 1.3.1. Sea A, B y C conjuntos:

(A ∩ B) ∪ C = (A ∪ C) ∩ (B ∪ C)

Explicamos lo que vamos a hacer.


Para probar esta igualdad, usamos el doble contenido.
Primeramente, veamos que (A ∩ B) ∪ C ⊆ (A ∪ C) ∩ (B ∪ C).
Damos la secuencia de razonamientos que expliquen este contenido
Sea x ∈ (A ∩ B) ∪ C. Tenemos dos posibilidades para x. x ∈ A ∩ B o x ∈ C. En el primer
caso, como x ∈ A ∩ B, debe ser x ∈ A y x ∈ B. Como A ⊆ A ∪ C y B ⊆ B ∪ C, debe ser
x ∈ A ∩ C y x ∈ B ∩ C. Por tanto x ∈ (A ∪ C) ∩ (B ∪ C). En el segundo caso, x ∈ C,
pero entonces, como C ⊆ C ∪ A y C ⊆ C ∪ B, tenemos que x ∈ C ∪ A y x ∈ C ∪ B. Por
lo que, de nuevo, x ∈ (A ∪ C) ∩ (B ∪ C).
Damos una conclusión que resuma lo que hemos visto.
En cualquier caso, hemos probado que para todo x ∈ (A ∩ B) ∪ C, x ∈ (A ∪ C) ∩ (B ∪ C),
por lo que (A ∩ B) ∪ C ⊆ (A ∪ C) ∩ (B ∪ C).
Veamos ahora el otro contenido (A∪C)∩(B∪C) ⊆ (A∩B)∪C. Sea x ∈ (A∪C)∩(B∪C).
Entonces x ∈ A ∪ C y x ∈ B ∪ C. Tenemos de nuevo dos posibilidades, o bien x ∈ C
o bien x ∈/ C. En el primer caso, si x ∈ C, también x ∈ (A ∩ B) ∪ C. Por otro lado, si
x∈ / C, como x ∈ A ∪ C, debe cumplirse que x ∈ A. De manera análoga, debe cumplirse
que x ∈ B y por tanto x ∈ A ∩ B. De donde, nuevamente, x ∈ (A ∩ B) ∪ C. Esto prueba
el segundo contenido.
Damos una conclusión que resuma lo que hemos visto.
Hemos probado que cada uno de los dos conjuntos contiene al otro, por lo que demostramos
que son iguales.

Ejercicio 1.3.2. Muestra dos conjuntos A, B tales que A\B = A, B\A = B. Para ese
conjunto A calcula un conjunto C tal que A\C = ∅ y C\A = ∅ ¿qué relación deben tener
A y C en este caso?

En este caso, si A\B = A eso significa que A y B no pueden tener elementos en


común. Si x ∈ A ∩ B, x pertenecerı́a a A pero no a A\B lo que contradice la hipótesis.
Es decir A ∩ B = ∅ y A y B son conjuntos disjuntos. Recı́procamente, si A y B son
disjuntos A\B = A. Observamos además que, si A\B = A, entonces B\A = B pues
ambas afirmaciones son equivalentes a A ∩ B = ∅. Por ejemplo, podemos tomar A = {1},
B = {2}.
Para la segunda parte, si A\C = ∅ significa que todo elemento de A es elemento de C,
por lo que A ⊆ C. Por otro lado, C\A = ∅ significa que C ⊆ A. Por tanto A = C y debe
ser C = {1} en nuestro ejemplo.

Ejercicio 1.3.3. Demuestra:

3. A′′ = A

11
1 Preliminares

Vamos a probar que A′′ ⊆ A y que A ⊆ A′′ . Primeramente, para probar que A′′ ⊆ A
vamos a ver que si x ∈ A′′ entonces x ∈ A. Para ello, si x ∈ A′′ = (A′ )′ , entonces por
definición de conjuntos complementario, x ∈/ A′ . Ahora bien, si x ∈ / A′ , debe ser x ∈ A.
De esta manera probamos que A ⊆ A. Veamos ahora el otro contenido. Tenemos que ver
′′

que si x ∈ A entonces x ∈ A′′ . Primeramente, si x ∈ A, por definición de complementario,


debe ser x ∈ / A′ . Nuevamente tomando el conjunto complementario, x ∈ A′′ y probamos
el otro contenido.

4. a) (A ∪ B)′ = A′ ∩ B ′

Esta es una de las leyes de De Morgan


http://es.wikipedia.org/wiki/Leyes de De Morgan
que ya sabemos que es cierta. De todas formas vamos a probarlo por doble contenido.
Para probar que (A ∪ B)′ ⊆ A′ ∩ B ′ tomemos un elemento x ∈ (A ∪ B)′ , se cumple que
x∈/ A ∪ B. Llegados a este punto, si x ∈ A entonces x ∈ A ∪ B. Luego si x ∈ / A ∪ B,
debe ser que x ∈ / A. De la misma manera, x ∈ / B. Es decir x ∈ A y x ∈ B , por lo que
′ ′

x ∈ A′ ∩ B ′ y tenemos el primer contenido.


Para probar el otro contenido, partimos de un elemento x ∈ A′ ∩ B ′ y queremos ver
que x ∈ (A ∪ B)′ . Puesto que x ∈ A′ ∩ B ′ entonces x ∈ A′ y x ∈ B ′ . Por definición de
complementario, se sigue que x ∈/Ayx∈ / B, de aquı́, como x no está en ninguno de los
dos conjuntos, no puede estar en la unión x ∈
/ A ∪ B, como no pertenece a este conjunto,
pertenecerá al complementario x ∈ (A ∩ B)′ .

10. Si A ⊆ B entonces B ′ ⊆ A′

Supongamos que A ⊆ B. Esto significa que todo elemento de A está en B, luego si x es


un elemento que NO está en B, entonces podemos asegurar que tampoco puede estar en
A. Usando sı́mbolos: Si x ∈
/ B entonces x ∈
/ A. Tomando la definición de complementario
de A y de B tenemos que si x ∈ B ′ entonces x ∈ A′ . Es decir B ′ ⊆ A′ .

12
2 El Principio de Inducción
2.1. Los Números Naturales
En este capı́tulo trabajamos con los números naturales y el principio de inducción.
Para ello introducimos los aximoas de Peano
Partimos de un conjunto N, llamado el conjunto de los números naturales, que cumple
los siguientes axiomas.

1. 0 ∈ N, el conjunto de los números naturales contiene un elemento especial llamado


cero.

2. Tenemos una función s : N → N llamada siguiente. Todo número natural tiene un


único siguiente.

3. N − s(N) = {0}, él único número natural que no es un siguiente de otro número
natural es el número 0. (∀x ∈ N, 0 ̸= s(x)).

4. s es inyectiva. Es decir, números naturales distintos tienen siguientes distintos. Si


x ̸= y, entonces s(x) ̸= s(y).

5. N cumple el principio de inducción.

El principio de inducción dice lo siguiente:


Sea X ⊆ N un subconjunto de los números naturales. Si X verifica:
1. 0 ∈ X

2. Si n ∈ X entonces s(n) ∈ X.
En estas condiciones X = N.
Observación 2.1.1. En algunos textos, el 0 no se considera un número natural. En los
axiomas anteriores sustituirı́amos el 0 por 1.
Normalmente, no escribimos s(x), sino x + 1, el siguiente de x. También denotamos
por 1 = s(0), 2 = s(1), 3 = s(2) etc.
El principio de inducción es útil para demostrar propiedades de los números naturales.
Sea P (n) una propiedad que depende de un número natural n. Si somos capaces de:

• Probar que la propiedad es cierta para el número 0.

• Probar que si k es un número natural que verfica la propiedad P , entonces somo


capaces de comprobar que k + 1 verifica la propiedad P .

13
2 El Principio de Inducción

Entonces, podemos afirmar que todos los números naturales verifican la propiedad P .
Ejemplo 2.1.2. Para todo número natural n
n(n + 1)
0 + 1 + 2 + 3 + ··· + n =
2

La suma de todos los números naturales entre 0 y n es n(n+1)


2 . Esta es una propiedad
que se puede cunmplir o no para cada número natural n. Por ejemplos, para el caso n = 7
7·8
0 + 1 + 2 + 3 + 4 + 5 + 6 + 7 = 28 =
2
y el número 7 verfica la propiedad.
Podemos usar el principio de inducción para demostrar que esta propiedad es cierta
cualquiera que sea el número n. Técnicamente definimos X = {n ∈ N | 0 + 1 + . . . + n =
n(n+1
2 } y usamos el principio de inducción. En la práctica analizamos simplemente la
propiedad
1. La propiedad es cierta si n = 0.
Efectivamente 0 = 0·12 , donde a la izquierda no hay que hacer ninguna suma.
2. Suponemos dado un número k que cumple la propiedad y debemos comprobar que
si siguiente k + 1 también cumple la propiedad.
En este ejemplo, suponemos que tenemos un número k que cumple:
k(k + 1)
0 + 1 + 2 + ... + k = (2.1)
2
y tenemos que comprobar que k + 1 también cumple la propiedad, es decir:
(k + 1)(k + 2)
0 + 1 + 2 + . . . + k + (k + 1) = (2.2)
2
Si queremos sumar todos los números entre 0 y k + 1, primeramente sumamos todos los
números entre 0 y k y sumamos a continuación el último. Pero, por hipótesis, hemos
supuesto que
k(k + 1)
0 + 1 + 2 + ... + k =
2
por lo que, sumando k + 1 a ambos lados
k(k + 1) k(k + 1) + 2(k + 1)
0 + 1 + 2 + . . . + k + (k + 1) = + (k + 1) = =
2 2
sacando factor común (k + 1)
(k + 1)(k + 2)
=
2
es decir, hemos probado que si la Ecuación 2.1 se cumple, también se cumplirá la
Ecuación 2.2.
El principio de inducción afirma que, en estas condiciones, la propiedad se cumple para
todo número natural.

14
2.2 Variantes de Inducción

Observación 2.1.3. En el ejercicio anterior, cuando escribimos

k(k + 1)
0 + 1 + 2 + ... + k =
2
podrı́a parecer que k > 2, pero esto es erróneo, lo único que sabemos que k es que es un
número natural que verifica la fórmula, podrı́a ser incluso que k = 0. Para evitar estas
ambigüedades usaremos más adelante otra notación para una suma de términos.

2.2. Variantes de Inducción


En ocasiones, el principio de inducción no se adecúa de manera inmediata a la propiedad
que queremos estudiar. Por ejemplo, podrı́a ocurrir que la propiedad solo es cierta para
los números n ≥ 5 o solo para los impares, para ello podemos ver variantes al principio
de inducción.

Ejemplo 2.2.1. Inducción a partir de un número a. En varias ocasiones, demostraremos


que algo es cierto a partir de n = 1.
Para probar que una propiedad es cierta para todos los naturales n ≥ a con a = ̸ 0
podemos simplemente:

1. Demostrar la propiedad para a.

2. Suponiendo que k ≥ a cumple la propiedad, demostrarla para k + 1.

Técnicamente demostrar que la propiedad P (n) es cierta para todo natural n ≥ a es


equivalente a demostrar que la propiedad Q(n) : P (n + a) es cierta para todo número
natural.

Ejemplo 2.2.2. Para todo n ≥ 4 tenemos que n2 − 1 > 3n. (Nótese que la propiedad no
es cierta para n = 0, 1, 2, 3).
Primeramente, la propiedad es cierta para n = 4, pues 42 − 1 = 15 > 12.
Ahora, suponemos que k ≥ 4 y k 2 − 1 > 3k, debemos probar que (k + 1)2 − 1 > 3(k + 1).

(k + 1)2 − 1 = k 2 + 2k
sabemos que, por hipótesis de inducción, k 2 > 3k + 1, por lo que

(k + 1)2 − 1 = k 2 + 2k > 3k + 2k + 1

como k ≥ 4, 2k + 1 ≥ 9 > 3

(k + 1)2 − 1 > 3k + 2k + 1 > 3k + 3 = 3(k + 1)

y se cumple la propiedad para k + 1. La propiedad queda demostrada por inducción.

De manera análoga se pueden hallar muchas variantes. Por ejemplo, para demostrar
que una propiedad P es cierta para todos los números impares, procederı́amos ası́:

15
2 El Principio de Inducción

1. 1 cumple la propiedad (el primer número impar).

2. Si k es un número impar que cumple la propiedad, tenemos que probar que el


siguiente número impar (k + 2) también cumple la propiedad.

Si P (n) es la propiedad que queremos demostrar para los números impares, realmente
harı́amos inducción sobre la propiedad Q(n) : P (2n + 1), pues todo número impar es de
la forma 2n + 1.

2.2.1. Inducción Completa


Esta variante es como sigue. Queremos probar la propiedad P

1. Probamos la propiedad para 0.

2. Suponemos que la propiedad se cumple para todos los números naturales {0, 1, . . . , k}
y, a partir de aquı́, probamos que es cierto para k + 1.

Este tipo de inducción es equivalente a hacer inducción normal en la sentencia alterna-


tiva
Q(n) : P es cierta para todos los k, 0 ≤ k ≤ n
En ocasiones, no es suficiente con conocer que la propiedad es cierta para un número k
para poder demostrar el caso k + 1, sino que necesitamos que la propiedad sea cierta
para números más pequeños que k. Veamos un ejemplo:

Ejemplo 2.2.3. Todo número natural n ≥ 2 es producto de números primos.


El caso base es n = 2, que es un número primo, luego es producto de (un) primos.
Si hiciéramos inducción directamente y suponemos que k es un producto de números
primos, esta información no es suficiente para demostrar que k + 1 se puede escribir como
producto de números primos, pues la factorización de k y de k + 1 no tiene nada que ver.
En este caso usamos inducción completa:
Supongamos que todos los números en el conjunto {2, 3, . . . , k} son producto de números
primos y vamos a deducir que k + 1 también lo es.

1. Si k + 1 es primo, ya está, k + 1 es producto de (un) primo.

2. Si k + 1 no es primo, eso significa que podemos escribir k + 1 = p · q con 1 < p < k + 1,


1 < q < k + 1, por la propia definición de primo. Por hipótesis de inducción
sabemos que p, q son productos de números primos. p = p1 · · · ps , q = q1 · · · qr .
Tendremos entonces que

k = p · q = p1 · · · ps q1 · · · qr

Es producto de números primos.

Por inducción completa, todo número natural n ≥ 2 se puede expresar como producto de
números primos.

16
2.3 Ejercicios de Inducción

2.3. Ejercicios de Inducción


Ejercicio 2.3.1. Demuestre por inducción:

i=0 (2 ∗ i + 1) = (n + 1)2
Pn

i=0 2 = 2n+1 − 1
Pn i

an+1 −1
= para a ̸= 1
Pn i
• i=0 a a−1

n(n+1)
=
Pn
• i=0 i 2

n(n+1)(2n+1)
=
Pn 2
• i=1 i 6

• 13 + 23 + 33 + · · · + n3 = (1 + 2 + · · · + n)2
n(n+1)(n+2)
+ 1) =
Pn
• i=1 i(i 3

3n(n+1)
=
P2n
• i=n i 2

=0
Pn
• i=−n i

n(3n+1)
=
P2n
• i=−n i 2

• Sea a ≥ −1 un número real. Demuestre por inducción que para todo natural
(1 + a)n ≥ 1 + an

Ejercicio 2.3.2. Busque una fórmula cerrada y demuestre por inducción:

• 1
1·2 + 1
2·3 + ··· + 1
n·(n+1)

Ejercicio 2.3.3. Demuestre por inducción:

= (n + 1)! − 1, n ≥ 1
Pn
• k=1 k(k!)

= (n − 1) · 2n+1 + 2, n≥1
Pn k
• k=1 k2

n(n+1)(2n+1)
= , n≥1
Pn 2
• k=1 k 6

Ejercicio 2.3.4. Demuestre por inducción:


n 1 1
• <2− , n≥2
P
i=1 i2 n

• n2 ≤ n!, n≥4

Ejercicio 2.3.5. Demuestre por inducción:

• n3 + 2n es múltiplo de 3 para todo natural n.

• n3 − n es siempre un múltiplo de 6.

17
2 El Principio de Inducción

• n2 − 1 es múltiplo de 8 para todo número natural impar.


• 7n − 2n es un múltiplo de 5.
• 7n − 1 es un múltiplo de 3.
Ejercicio 2.3.6. Sea A un conjunto, llamamos partes de A al conjunto P (a), que tiene
como elementos los subconjuntos de A.
x ∈ P (A) ↔ x ⊆ A
Ası́, P (∅) = {∅} tiene un elemento y
P ({a, b, c}) = {∅, {a}, {b}, {c}, {a, b}, {a, c}, {b, c}, {a, b, c}})
tiene 8 elementos.
Si A tiene n elementos ¿Cuantos tendrá P (A)? Demuestre el número de elementos que
tiene P (A) usando inducción.
Ejercicio 2.3.7. Si P es un polı́gono del plano, una diagonal es un segmento que una
dos puntos que no estén en el mismo lado. Ası́, un triángulo no tiene diagonales y un
cuadrado tiene dos diagonales.
Demuestre que el número de diagonales de un polı́gono con n lados n ≥ 3 es n(n−3)
2 .
Ejercicio 2.3.8. Dado un conjunto de rectas r1 , . . . , rs . Diremos que están en posición
general si:
• No hay dos rectas iguales.
• Dos rectas cualesquiera del conjunto se cortan en un punto (no hay paralelas).
• No hay tres rectas que pasen por el mismo punto.
Demuestre, usando inducción, que n rectas (n ≥ 2) en posición general, se cortan en
n2 −n
2 puntos.
Ejercicio 2.3.9. Considera el código

def ordena_bruto(lista):
if len(lista) < 2:
return lista
sublista1 = lista[:-1]
ultimo = lista[-1]
listaordenada = ordena_bruto(sublista1)
if listaordenada[-1] <= ultimo:
return listaordenada + [ultimo]
else:
nuevoultimo = listaordenada[-1]
listaordenada[-1] = ultimo
listaordenada = ordena_bruto(listaordenada)
return listaordenada + [nuevoultimo]

18
2.3 Ejercicios de Inducción

Usa inducción para probar que:

• El algoritmo siempre termina.

• El algoritmo devuelve una lista con los mismos elementos que el input pero ordenada
de manera creciente.

• El algoritmo hace, a lo sumo n(n + 2)/2 llamadas recursivas a ordena bruto

Este método de ordenación es una implementación recursiva del llamado método de la


burbuja.

Ejercicio 2.3.10. Busque el error de la siguiente “demostración”por inducción completa


de que
an = 1 ∀n ≥ 0∀a ∈ R − {0}

1. Primero, para n = 0, a0 = a1−1 = a


a =1

2. Ahora, supuesto cierto para todo m ≤ n, veamos que es cierto para n + 1. Tenemos
a2n an an 1·1
que an+1 = a2n−(n−1) = n−1 = n−1 = = 1.
a a 1

2.3.1. La Sucesión de Fibonacci


Consideremos la función

f: N → N 

 0 Si x = 0
x 7→ f (x) = 1 Si x = 1
f (x − 1) + f (x − 2)

En otro caso

Calcula los primeros valores de f .


A la sucesión (f (0), f (1), f (2), . . .) se le llama la sucesión de Fibonacci.
Una función Sage recursiva que calcula la función de Fibonacci es:

def Fibo(n):
if (n == 0) or (n == 1):
return n
return Fibo(n-1) + Fibo(n-2);

Ejercicio 2.3.11. Demuestre por inducción que el código Sage termina en una cantidad
finita de pasos para cada número natural n.

Ejercicio 2.3.12. Sea ϕ el número áureo. y ψ = 1 − ϕ = 1− 5
2

1+ 5
ϕ=
2

19
2 El Principio de Inducción

Demuestre que ϕ y ψ verifican la recursión de Fibonacci, es decir, para todo n

ϕn + ϕn+1 = ϕn+2

ψ n + ψ n+1 = ψ n+2
Utilice este hecho para demostrar por inducción que, para todo n
ϕn − ψ n
F ib(n) = √
5
Nota: de la anterior fórmula vemos que F ib(n) crece exponencialmente. En particular,
crece más rápido que cualquier potencia.
F ib(n)
∀k > 0 lı́m =∞
n→∞ nk
Ejercicio 2.3.13. Demuestre por inducción que, para todo n ≥ 2, para calcular F ib(n)
el código Sage dado necesita realizar F ib(n + 1) − 1 sumas. ¿Qué opina sobre el coste
en número de operaciones que necesita este algoritmo? ¿Cree que hay alguna forma de
calcularlo más rápido?
Ejercicio 2.3.14. Sea A la matriz
!
0 1
A=
1 1

Pruebe que para todo n ≥ 1


!
F ib(n − 1) F ib(n)
An =
F ib(n) F ib(n + 1)

Utilice esta matriz para probar que

F ib(2n) = F ib(n)(F ib(n − 1) + F ib(n + 1))

Esta propiedad de la matriz A permite generar un código más rápido para calcular
F ib(n).

def potencia_matr_rapida(A, n):


if n == 0:
raise ValueError(’the exponent must be strictly positive’)
if n == 1:
return A;
if is_odd(n):
return multiplica_matriz(A, potencia_matr_rapida(A,n-1))
if is_even(n):
B = potencia_matr_rapida(A,n//2)
return multiplica_matriz(B, B)

20
2.3 Ejercicios de Inducción

def Fiborapido(n):
if (n == 0) or (n == 1):
return n;
A = matrix([[0,1],[1,1]])
return potencia_matricial_rapida(A,n)[0][1];

Ejercicio 2.3.15. Tomemos un tablero de casillas n × 2 como en la figura

Caso n = 6

Tenemos piezas de dominó de tamaño 1 × 2, con ellas podemos rellenar el tablero de


muchas maneras distintas.

Una forma de rellenar el tablero n = 6.

Demuestra, usando inducción, que el tablero n × 2 tiene F ib(n + 1) formas distintas de


rellenarse con fichas de dominó.

2.3.2. Ejercicio de Inducción Resuelto


Veamos como se resolverı́a un ejercicio de inducción de una manera completa y detallada.
Hagamos el siguiente ejercicio de la hoja de problemas.

Ejercicio 2.3.16.

13 + 23 + 33 + · · · + n3 = (1 + 2 + · · · + n)2 (⋆)

La suma de los n primeros cubos es igual al cuadrado de la suma de los n primeros


enteros positivos.
▶ Como primer paso, vamos a demostrar que la fórmula es cierta para n = 1. Tenemos
que comprobar que 13 = 12 lo cual es trivialmente cierto.
▶ Ahora vamos aplicar el paso inductivo en el cual vamos a suponer que, para un
número natural n, la formula (⋆) es cierta y vamos a probar que la fórmula es cierta para
n + 1. Es decir, suponemos que se cumple que

13 + 23 + 33 + . . . + (n − 1)3 + n3 = (1 + 2 + 3 + . . . + (n − 1) + n)2
Y, usando esta igualdad, debemos probar que:

21
2 El Principio de Inducción

13 + 23 + . . . + n3 + (n + 1)3 = (1 + 2 + 3 + . . . + n + (n + 1))2
Partamos de la parte de la izquierda de la igualdad y veamos si podemos transformarlo
en el elemento de la derecha.

13 + 23 + 33 + . . . + n3 + (n + 1)3 (2.3)
La hipótesis de inducción nos da una fórmula para la suma de los n primeros cubos,
luego sustituimos (13 + 23 + . . . + n3 ) por (1 + 2 + . . . + n)2 en 2.3, nos queda

13 + 23 + 33 + . . . + n3 + (n + 1)3 = (1 + 2 + . . . + n)2 + (n + 1)3

Tenemos que ver si esta cantidad es la misma que

(1 + 2 + 3 + . . . + n + (n + 1))2

Si operamos, tenemos que 2.3.2 vale

(1 + 2 + . . . + n)2 + (n + 1)2 + 2(1 + 2 + . . . + n)(n + 1)

Ahora, hemos visto en un ejercicio anterior que (1 + 2 + . . . + n) = n(n+1)


2 , luego la última
cantidad vale
n(n + 1)
(1 + 2 + . . . + n)2 + (n + 1)2 + 2 (n + 1) =
2
(1 + 2 + . . . + n)2 + (n + 1)2 + n(n + 1)2
Sacando factor común en el término (n + 1) nos queda

(1 + 2 + . . . + n)2 + (n + 1)2 · (1 + n) = (1 + 2 + . . . + n)2 + (n + 1)3

Tenemos por tanto que

13 + . . . + n3 + (n + 1)3 = (1 + 2 + . . . + n)2 + (n + 1)3 = (1 + . . . + n + (n + 1))2

Que es lo que querı́amos demostrar.


Puesto que la igualdad se cumple para n = 1 y, siempre que se cumple para n tenemos
que se cumple para n + 1. Por el principio de inducción, la propiedad es cierta para todo
n ≥ 1.

22
3 Espacios Vectoriales
En esta parte de la asignatura, la estructura matemática que queremos estudiar es la
de espacio vectorial. Son conocidos los ejemplos de R2 y R3 , pero los espacios vectoriales
son mucho más generales y la teorı́a se puede aplicar en contextos diversos.
Asociado a cada espacio vectorial V tenemos siempre un cuerpo K (las constantes).
Los tipos de objetos que vamos a trabajar son dos:

1. Los elementos de V , los llamaremos vectores v1 = (2, 3, 4), v2 = (3, 5, 4)

2. Los elementos de K, los llamaremos constantes, λ = 3 ∈ Q.

• podemos sumar vectores y el resultado es un vector v1 + v2 = (5, 8, 8).

• podemos multiplicar una constante por un vector, el resultado será un vector


λ · v1 = (6, 9, 12).

2⃗v 2⃗v
⃗u +

w =3
⃗v

⃗u 3⃗u

La definición formal es:

Definición 3.0.1. Un cuerpo K es un conjunto con al menos dos elementos 0 ̸= 1 y dos


operaciones +, ∗ que cumplen las siguientes propiedades, para a, b, c ∈ K:

• Conmutativa para la suma a + b = b + a

• Asociativa para la sumaa + (b + c) = (a + b) + c

• Elemento neutro de la suma 0 + a = a

• Dado a, existe un elemento opuesto −a tal que a + (−a) = 0.

• Conmutativa para el producto a ∗ b = b ∗ a

• Asociativa para el producto a ∗ (b ∗ c) = (a ∗ b) ∗ c

• Elemento neutro del producto 1 ∗ a = a

• Dado a ̸= 0, existe un elemento inverso a−1 tal que a ∗ a−1 = 1.

23
3 Espacios Vectoriales

• Propiedad discributiva a ∗ (b + c) = a ∗ b + a ∗ c.
Ejemplo 3.0.2. Son cuerpos: Q, R, C. Si tomamos el conjunto Z2 = {0, 1}, este conjunto
con las operaciones:
+ 0 1 ∗ 0 1
0 0 1 0 0 0
1 1 0 1 0 1
también es un cuerpo.
Definición 3.0.3. Un conjunto V se dice que es un espacio vectorial sobre un cuerpo
K si existen dos operaciones:

+: V ×V → V ∗: K×V → V
(u, v) 7→ u + v (λ, u) 7 → λ∗u

suma de vectores y multiplicación de un vector por un escalar, que cumple: si u, v, w ∈ V


y λ, µ ∈ K:
• u+v =v+u

• u + (v + w) = (u + v) + w

• existe un elemento 0 (vector cero) tal que 0 + u = u + 0 = u

• Dado un vector u existe otro vector v con u + v = 0

• λ ∗ (u + v) = λ ∗ u + λ ∗ v

• (λ +K µ) ∗ u = λ ∗ u + µ ∗ u

• λ ∗ (µ ∗ u) = (λ ·K µ) ∗ u

• 1K ∗ u = u
A los elementos de V los denominados vectores, a los elementos de K escalares (o
constantes).
Ejemplo 3.0.4. 1. R2 es un R-espacio vectorial con las operaciones

(a, b) + (c, d) = (a + c, b + d) λ ∗ (a, b) = (λa, λb)

0 = (0, 0), −(a, b) = (−a, −b)

2. De la misma manera, Rn (las tuplas con n componentes) también es un R-espacio


vectorial.

(a1 , . . . , an ) + (b1 , . . . , bn ) = (a1 + b1 , . . . , an + bn )


λ ∗ (a1 , . . . , an ) = (λa1 , . . . , λan )

24
3. C 0 ([0, 1]) = {f : [0, 1] → R | f continua }.
Es un espacio vectorial sobre R donde la suma de vectores es:

f + g : [0, 1] → R
x 7→ f (x) + g(x)

El producto por un escalar es la función

λ · f : [0, 1] → R
x 7→ λ · f (x)

4. C es un R-espacio vectorial.
La suma viene dada por la suma de números complejos y el producto de un número
real por un número complejo es el producto usual.

5. R es un Q-espacio vectorial con el producto y la suma en R.

6. R[x]<3 los polinomios con coeficientes reales de grado menor que 3 forman un
R-espacio vectorial.

7. Las matrices m×n con coeficientes racionales, Mm×n (Q) forman un espacio vectorial
sobre Q.

8. Tomemos el conjunto Π = {(a, b, c) ∈ R3 | a + b − c = 0}.


Π es un subconjunto del espacio vectorial R3 .
Además, Π es un espacio vectorial con las mismas operaciones de suma y producto
por constante de R3 ,
• Si (a, b, c), (d, e, f ) ∈ Π, a + b − c = 0, d + e − f = 0 (a + d) + (b + e) − (c + f ) = 0
y (a + d, b + e, c + f ) ∈ Π
• Si (a, b, c) ∈ Π, a + b − c = 0, λa + λb − λc = 0 y λ(a, b, c) ∈ Π

Definición 3.0.5. Un vector u ∈ V es combinación lineal de {v1 , . . . , vr } ⊆ V si


existen a1 , . . . , ar ∈ K tales que

u = a1 v1 + . . . + ar vr

Ejemplo 3.0.6. En R2 , el vector (3, 2) es combinación lineal de (1, 0) y (1, 1) pues

(3, 2) = (1, 0) + 2(1, 1)

En R3 , el vector (1, 0, 1) no es combinación lineal de (1, 1, 0) y (1, 0, 2), pues para ser
combinación lineal, deberı́an existir dos constantes a, b de manera que

(1, 0, 1) = a(1, 1, 0) + b(1, 0, 2)

25
3 Espacios Vectoriales

pero entonces, a y b deben cumplir:



a +
 b = 1
a = 0
2b = 1

que es un sistema incompatible.


Definición 3.0.7. Un conjunto de vectores {v1 , v2 , . . . , vr } es ligado o linealmente
dependiente si existen λ1 , . . . , λr no todos cero tales que
λ1 v1 + . . . + λr vr = 0
Teorema 3.0.8.
Si en una familia tenemos el vector cero, es automáticamente ligada.
Si en una familia tenemos vectores repetidos, es automáticamente ligada.
Definición 3.0.9. Un conjunto de vectores {v1 , . . . , vr } es libre o linealmente inde-
pendiente si no es ligado.
Un conjunto es libre si y solo si
λ1 v1 + . . . + λr vr = 0 −→ λ1 = . . . = λr = 0
(la única solución es la nula).
Para detectar si una familia de vectores {v1 , . . . , vr } es linealmente dependiente o
independiente, debemos comprobar cuales son las soluciones de:
λ1 v1 + . . . + λr vr = 0
Este es un sistema lineal en las incógnitas λ1 , . . . , λr que es homogéneo, luego siempre es
compatible. Si es determinado, la única solución es λ1 = . . . = λr = 0 y los vectores son
linealmente independientes. Si el sistema es indeterminado, habrá soluciones distintas de
cero y los vectores serán linealmente dependientes.
Ejemplo 3.0.10. Veamos si, en R3 , los vectores (1, 0, 1), (0, 1, −1), (2, 1, 1) son linealmente
independientes o no. Tenemos que comprobar cuales de qué manera podemos escribir el
vector (0, 0, 0) como combinación lineal de estos vectores. Buscamos los valores posibles
de a, b y c para que:
a(1, 0, 1) + b(0, 1, −1) + c(2, 1, 1) = (0, 0, 0)
Resulta el sistema: 
a
 + 2c = 0
b + c = 0
a − b + c = 0

Resolviendo el sistema por Gauss, obtenemos que las soluciones son:


a = −2t b = −t c=t
Los tres vectores son linealmente dependientes, por ejemplo, para t = 1
−2(1, 0, 1) − (0, 1, −1) + (2, 1, 1) = (0, 0, 0)

26
Ejemplo 3.0.11. Veamos si, en R3 , los vectores (1, 0, 1), (0, 1, −1), (0, 1, 1) son lineal-
mente independientes o no. Buscamos valores para que:

a(1, 0, 1) + b(0, 1, −1) + c(0, 1, 1) = (0, 0, 0)

Analizamos el sistema: 
a
 = 0
b + c = 0
a − b + c = 0

Este sistema es compatible determinado, la única solución es a = 0, b = 0, c = 0. Por


tanto, los tres vectores dados forman una familia libre.

Teorema 3.0.12. Sea S = {v1 , . . . , vr } ⊆ V

• Si S es libre, entonces cualquier subconjunto de S es libre.

• Si S es ligada, y S ⊆ T entonces T es ligada.

• Si S es ligada, hay un vector u ∈ S que es combinación lineal de los restantes


S − {u}.

• Si S es libre y u ∈
/ ⟨S⟩ entonces S ∪ {u} es libre.

• {u, v} es ligada si uno de los vectores es múltiplo del otro.

Definición 3.0.13. Una matriz n × m sobre K es una tabla con n filas y m columnas
cuyos elementos son elementos de K.

a11 . . . a1m
 

A= ..
.
 

an1 . . . amn

denotaremos por aij el término de la fila i y columna j de A. Una matriz es cuadrada si


el número de filas es igual al de columnas.

Definición 3.0.14. Dadas dos matrices A y B de tamaño n × m, la suma C = A + B


es la matriz n × m tal que cij = aij + bij . Si A es una matriz n × m y B es una matriz
m × p, el producto C = AB es la matriz n × p tal que:

cij = ai1 b1j + ai2 b2j + . . . + aip bpj

La matriz identidad In es la matriz n × n cuyas entradas son 1 en la diagonal principal y


0 fuera de ella. Verifica que si A es n × m, In A = A, AIm = A.

Definición 3.0.15. Dada una matriz A, su rango es el número máximo de filas li-
nealmente independientes. Este número coincide con el número máximo de columnas
linealmente independientes.

27
3 Espacios Vectoriales

Definición 3.0.16. Dada una matriz A, son operaciones elementales por filas
(columnas):
• Intercambiar dos filas (columnas)
• Sumar a una fila (columna) un múltiplo de otra.
• Multiplicar una fila (columna) por una constante c ̸= 0.
Las operaciones elementales por filas o columnas aplicadas a una matriz no cambia su
rango.
Definición 3.0.17. Dada una fila no nula de una matriz, llamamos pivote al primer
término (leı́da la fila de izquierda a derecha) distinto de cero. Una matriz es escalonada
por filas si:
• Las filas nulas son las últimas filas de la matriz.
• En cada fila no nula, el pivote está estrictamente a la derecha del pivote de la fila
anterior.
Si A es una matriz escalonada por filas, su rango es igual al número de filas no nulas.
0 1 1
 

Ejemplo 3.0.18. La matriz: 0 1 −1 no es escalonada.


 
0 0 3
1 0 −1 3
 

La matriz: 0 0 3 −1 es escalonada.


 
0 0 0 0
Teorema 3.0.19 (eliminación gausiana). Dada una matriz A, podemos calcular una
matriz escalonada B a partir de A aplicando operaciones elementales por filas.
Definición 3.0.20. Consideremos un sistema de ecuaciones lineales:

 a11 x1
 + a12 x2 + . . . + a1m xm = b1
...
+ an2 x2 + . . . + anm xm = bn

a x
n1 1

con n ecuaciones y m incógnitas. El sistema es:


• Incompatible: si no tiene soluciones.
• Compatible determinado: si tiene una única solución.
• Compatible indeterminado: si tiene más de una solución.
La matriz asociada y la matriz ampliada del sistema son:
a11 . . . a1m a11 . . . a1m b1
   
.. .. .. 
. . . 
  
  
an1 . . . anm an1 . . . anm bn

28
Teorema 3.0.21 (Rouché-Frobenius). Un sistema de ecuaciones es compatible si y solo
si el rango de la matriz asociada es igual al rango de la matriz ampliada. En este caso,
es determinado si el rango es igual al número de incógnitas.

Teorema 3.0.22. Podemos calcular todas las soluciones de un sistema de ecuaciones


lineales mediante la eliminación gausiana.

Definición 3.0.23. Una matriz n × n A es invertible si existe una matriz inversa A−1
tal que AA−1 = A−1 A = In .

Teorema 3.0.24. Sean A, B matrices n × n. A es invertible si y solo si tiene rango n.


Si A y B son invertibles, AB también lo es y (AB)−1 = B −1 A−1 . Podemos calcular la
inversa de una matriz mediante el método de Gauss-Jordan.

Teorema 3.0.25. tenemos una función | · | : Mn×n (K) → K que a cada matriz A n × n le
asocia un número real |A| llamado determinante que verifica las siguientes propiedades:

• El determinante de la matriz identidad es 1.

• Si intercambiamos dos filas (o dos columnas) de A, el determinante cambia de signo.

• Si a una fila (columna) de A le sumamos un múltiplo de otra, el determinante no


cambia.

• Si a una fila (columna) de A la multiplicamos por una constante c el determinante


queda multiplicado por c.

Ejemplo 3.0.26. Para matrices 3 × 3, el determinante viene dado por la regla de Sarrus:

a a12 a13
11
a21 a22 a23 =


a31 a32 a33

= a11 a22 a33 + a12 a23 a31 + a21 a32 a13 − a31 a22 a13 − a32 a23 a11 − a21 a12 a33

Teorema 3.0.27. Si A y B son matrices n × n.

• A es invertible si y solo si |A| =


̸ 0. Además |A−1 | = |A|−1 .

• |A| = |At |.

• |AB| = |A| · |B|

• (desarrollo por una fila) Si denotamos Aij la matriz que resulta de eliminar la fila i
y columna j, el determinante se puede calcular por desarrollo de la fila (columna) k.

|A| = (−1)k+1 |Ak1 | + (−1)k+2 |Ak2 | + . . . + (−1)k+n |Akn |

De la definición de rango, tenemos lo siguiente:

29
3 Espacios Vectoriales

Ejemplo 3.0.28. Sean {v1 , . . . , vr } vectores de Kn . Podemos construir la matriz M


de r y n columnas donde, en la fila i, escribimos los coeficientes de vi . Entonces son
equivalentes:

• {v1 , . . . , vr } es libre.

• M tiene rango r.

Ejemplo 3.0.29. Si tenemos en R4 los vectores u = (a, b, c, d), v = (e, f, g, h), w =


(i, j, k, l). Construimos la matriz
 
a b c d
M =  e f g h
 
i j k l

Los vectores {u, v, w} son linealmente independientes si y solo si la matriz M tiene rango
3.

30
4 Subespacios
4.1. Subespacio vectorial
Definición 4.1.1. Si V es un espacio vectorial sobre K y W ⊆ V un subconjunto no
vacı́o, diremos que W es un subespacio si es un espacio vectorial con las operaciones de
V.

Teorema 4.1.2. Si W ⊆ V , W es un subespacio si, W ̸= ∅ y si v, w ∈ W , a, b ∈ K


entonces av + bw ∈ W .

Definición 4.1.3. Si S = {v1 , . . . , vr } ⊆ V es un conjunto de vectores de V , definimos

⟨S⟩ = {λ1 v1 + . . . + λr vr | λ1 , . . . , λn ∈ K}

El conjunto de todas las combinaciones lineales de v1 , . . . , vr .

Este conjunto W = ⟨S⟩ es siempre un subespacio de V , lo llamamos el subespacio


generado por S. También diremos que la expresión

λ1 v1 + . . . + λr vr

son ecuaciones paramétricas de ⟨S⟩ y que {v1 , . . . , vr } es un conjunto generador


de ⟨S⟩.

Ejemplo 4.1.4.

• Rn está generado por los vectores e1 = (1, 0, . . . , 0), e2 = (0, 1, . . . , 0),. . ., en =


(0, 0, . . . , 1). Cualquier vector es combinación de los vectores ei .
n
(a1 , . . . , an ) =
X
ai · ei
i=1

• Q[x]<4 está generado por ⟨1, x, x2 , x3 ⟩ pero también por ⟨1, 1 + x, 1 + x2 , 1 + x +


x3 , 1 − x + x3 ⟩.

• El conjunto U = {(b + c, a + c, a + b + d) | a, b, c, d ∈ R} es un espacio dado de


manera paramétrica por

U = ⟨(0, 1, 1), (1, 0, 1), (0, 1, 1), (0, 0, 1)⟩

En este caso tenemos que U = R3

31
4 Subespacios

• En R3 el conjunto
U = {x + 3y − 4z = 0}
es un espacio vectorial dado de manera implı́cita.
Por otro lado el conjunto
V = {(λ, λ, λ) | λ ∈ R}
W = {(λ + 5µ, λ + µ, λ + 2µ) | λ, µ ∈ R}
son espacios vectoriales dados de manera paramétrica V = ⟨(1, 1, 1)⟩, W = ⟨(1, 1, 1), (5, 1, 2)⟩.
Sea V = Kn espacio vectorial sobre K.
Si tenemos un conjunto de ecuaciones homogéneas

a11 x1
 + a12 x2 + . . . + a1n xn = 0
...
+ at2 x2 + . . . + atn xn = 0

a x
t1 1

El conjunto de soluciones W del sistema es siempre un subespacio vectorial de V . A estas


ecuaciones se les llama ecuaciones implı́citas de W .
Teorema 4.1.5. Todo subespacio de Kn se puede escribir de manera implı́cita o pa-
ramétrica. En particular, los subespacios vectoriales de Kn son las soluciones de los
sistemas de ecuaciones homogéneos con n variables.

4.1.1. Paso de la representación implı́cita a generadores


Para calcular las ecuaciones paramétricas de un espacio dado de manera implı́cita,
simplemente calculamos las soluciones.
Ejemplo 4.1.6. En R4 vamos a calcular generadores del subespacio:

U = {(x, y, z, t) ∈ R4 | x + 2y = 0, y − t = 0}

Resolvemos el sistema de ecuaciones homogéneas dado, la matriz de coeficientes es:


!
1 2 0 0
0 1 0 −1

Esta matriz está ya escalonada, los pivotes se corresponden con las variables x, y, por
tanto z y t serán parámetros. Resolviendo:

= −2b


 x
=

y b

 z = a
t =


b

Para calcular generadores, escogemos cada una de las variables, la sustituimos por 1 y el
resto por cero.
U = ⟨(0, 0, 1, 0), (−2, 1, 0, 1)⟩

32
4.1 Subespacio vectorial

4.1.2. Paso de la representación por generadores a implı́citas


Vamos a ver dos métodos distintos válidos para calcular las ecuaciones implı́citas de
un subespacio de Kn dado por generadores. Para ambos métodos recomiendo leer en
paralelo el caso general y el Ejemplo 4.1.7
Primer método:
Si nos dan un subespacio de Kn

V = ⟨v1 = (a11 , . . . , a1n ), . . . , vr = (ar1 , . . . , arn )⟩


buscamos ecuaciones de manera que los vectores de V sean sus soluciones.
Una ecuación implı́cita con coeficientes α1 , . . . , αn tiene la forma:

α1 x1 + . . . + αr xr = 0

pero, si v1 es solución, entonces se tiene que cumplir:

α1 a11 + . . . + αn a1n = 0

igualmente, para cada vector vi

α1 ai1 + . . . + αn ain = 0

Esto nos da un sistema, expresado matricialmente como:

α1 0
    
a11 . . . a1n
  ..   .. 
...   .  = .


ar1 . . . arn αn 0

resolviéndolo nos quedará que las soluciones son:

α1 = f1 (t1 , . . . , ts )
..
.
αn = fn (t1 , . . . , ts )

dependientes de s parámetros t1 , . . . , ts .
Para cada parámetro obtendremos una ecuación implı́tica. Por ejemplo, para t1 = 1 y
el resto cero, la solución:
α1 = f1 (t1 , 0, . . . , 0)
..
.
αn = fn (t1 , 0, . . . , 0)
nos proporciona la ecuación (sustituyendo las α en la ecuación general

f1 (t1 , 0, . . . , 0)x1 + · · · + fn (t1 , 0, . . . , 0)xn = 0

En total tendremos s ecuaciones implı́citas, una por cada parámetro.

33
4 Subespacios

Segundo método:
Si nos dan un subespacio de Kn

V = ⟨v1 = (a11 , . . . , a1n ), . . . , vr = (ar1 , . . . , arn )⟩

equivalentemente:
 x1 = a11 t1 + a21 t2 + . . . + ar1 tr

..

V = .

xn = a1n t1 + a2n t2 + . . . + arn tr

Escribimos estos coeficientes y las variables en una matriz

a11 ··· ar1 x1


 
..
.
 
 
a1n ··· arn xn

Ahora, aplicamos Gauss hasta escalonar la parte de los coeficientes. las últimas s filas
serán ceros en la parte de los coeficientes. Algo de la forma:

* ··· * *
 
..
.
 
 
 
 * ··· * * 
 
 0 ··· 0 g1 (x1 , . . . , xn ) 
 
..
 
.
 
 
0 ··· 0 gs (x1 , . . . , xn )

De la última columna, los polinomios que aparecen en filas que el resto de coeficientes
sean cero, forman ecuaciones implı́citas de V .

g1 (x1 , . . . , xn ) = 0

V = ···
g (x , . . . , x )

= 0
s 1 n

Ejemplo 4.1.7. En R4 consideremos:

V = ⟨(2, 0, 1, 1), (2, 1, −1, 0)⟩

equivalentemente:
= 2a + 2b


x
=

y b
V =

 z = a − b
t =


a
Primer método:

34
4.1 Subespacio vectorial

Buscamos ecuaciones de la forma αx + βy + γz + δt = 0. Sabemos que (2, 0, 1, 1) y


(2, 1, −1, 0) tienen que ser solución luego
(
2α + γ + δ = 0
2α + β − γ = 0
resolviendo el sistema tenemos que las soluciones son:
= −u/2 − v/2


 α
= 2u + v

β

 γ = u
=


δ v
Cada uno de los parámetros nos da una ecuación:
Para u = 1, v = 0, (α, β, γ, δ) = (−1/2, 2, 1, 0), que nos da la ecuación:
−1/2x + 2y + z = 0
Para u = 0, v = 1, (α, β, γ, δ) = (−1/2, 1, 0, 1), que nos da la ecuación:
−1/2x + y + t = 0
Por lo que las ecuaciones implı́citas de V son:
(
−1/2x + 2y + z = 0
V =
−1/2x + y + t = 0
Segundo método: De la representación de V escribimos la matriz:
2 2 x
 

 0 1 y 

1 -1 z
 
 
1 0 t
y escalonamos por Gauss
2 2 x 1 0 t 1 0 t
     
−
0 1 y 0 1 y  F−3−−−−−→  0 1 y
  −−−−−→  
−F1 →F3 
 F1 ↔ F4 
   
1 -1 z 1 -1 z 0 -1 z-t
  F4 −2F 1 →F4 
 
    
1 0 t 2 2 x 0 2 x-2t
1 0 t
 
−− −−− −−→
F3 +F2 →F3
 0 1
 y 

F4 −2F2 →F4  0

0 y+z-t 

0 0 x-2y-2t
De aquı́, las ecuaciones implı́citas de V son la ecuación que aparece en la última columna
de las filas que sean cero:
(
y +z −t = 0
V =
x −2y −2t = 0
Los dos métodos nos dan resultados diferentes, pero ambos son correctos ya que la
representación no es única.

35
5 Bases, dimensión, Fórmula de Grassmann
Definición 5.0.1. Sea V un espacio vectorial, un subconjunto S es un conjunto
generador de V si V = ⟨S⟩. Una base de V es una familia de vectores que es libre y
conjunto generador.

Teorema 5.0.2. Todas las bases de U tienen el mismo número de elementos, la dimen-
sión de U .

Ejemplo 5.0.3. En R2 , B = [(1, 0), (0, 1)] es una base. Es un conjunto generador, pues

(x, y) = x(1, 0) + y(0, 1)

Además es libre, pues si


(0, 0) = x(1, 0) + y(0, 1)
debe ser x = y = 0. Como la base tiene 2 elementos, R2 tiene dimensión 2. En general
dim(Kn ) = n como espacio vectorial sobre K.

Ejemplo 5.0.4. En R3 , A = [(1, 1, 0), (1, −1, 0), (0, 0, 1)] es una base. Los vectores son
independientes, pues:
1 1 0
 

rango 1 −1 0 = 3
 
0 0 1
Es un conjunto generador, pues dado cualquier vector (a, b, c), el sistema de ecuaciones
en las incógnitas x, y, z

x(1, 1, 0) + y(1, −1, 0) + z(0, 0, 1) = (a, b, c)

siempre tiene solución. la dimensión de R3 es 3.


Por otro lado B = [(1, 1, 0), (0, 0, 1)] es un conjunto libre, pero no es base, pues no es
generador. Por ejemplo:
(1, 0, 0) ∈/ ⟨(1, 1, 0), (0, 0, 1)⟩
C = [(1, 0, 0), (0, 1, 0), (0, 0, 1), (1, 2, 3)] es un conjunto generador, cualquier vector de
R3 es combinación lineal de los vectores de C, pero no es base, ya que no es libre:

1(1, 0, 0) + 2(0, 1, 0) + 3(0, 0, 1) − (1, 2, 3) = (0, 0, 0)

Definición 5.0.5. Sea V = Kn sobre K. Entonces V tiene dimensión n y tenemos una


base canónica:
V = ⟨(1, 0, . . . , 0), (0, 1, 0, . . . , 0), . . . , (0, . . . , 0, 1)⟩

37
5 Bases, dimensión, Fórmula de Grassmann

Llamamos ei al i-ésimo vector de la base canónica, que tiene un uno en la posición i y


ceros en el resto. Ası́, la base canónica de Q4 es

Bc = [e1 , e2 , e3 , e4 ] = [(1, 0, 0, 0), (0, 1, 0, 0), (0, 0, 1, 0), (0, 0, 0, 1)]

Ejemplo 5.0.6. Sea R[x]<3 los polinomios en la variable x que tienen grado menor que
3.
R[x]<3 = {a + bx + cx2 }
Tenemos que los polinomios [1, x, x2 ] forman una base del espacio ¿Por qué?. Por tanto
R[x]<3 tiene dimensión 3.

Teorema 5.0.7.

• Si T es un conjunto generador de V , entonces T contiene una base.

• Si B = [v1 , . . . , vn ] es una base de V y S es un conjunto libre de r elementos de V ,


entonces existen n − r elementos de B vi1 , . . . , vin−r tales que S ∪ {vi1 , . . . , vin−r } es
una base de V .

Ejemplo 5.0.8. Sea

T = [(2, 1, 0, 2), (3, 4, 2, 1), (1, 3, 2, −1), (3, 4, 2, 1), (0, 0, 1, −1), (0, 0, 0, 0)]

y V = ⟨T ⟩. V es el subespacio de R4 consistente en todas las combinaciones lineales de


elementos de T . Calculemos una base de V y su dimensión.
Primeramente, podemos eliminar los vectores cero y los repetidos. Ası́

V = ⟨(2, 1, 0, 2), (3, 4, 2, 1), (1, 3, 2, −1), (0, 0, 1, −1)⟩

Vamos buscando vectores independientes en la lista de generadores:

rango(2 1 0 2) = 1

Tomamos este vector y lo añadimos en una lista

L = [(2, 1, 0, 2)]

tomamos el siguiente generador y lo añadimos a la matriz formada por los vectores de


L. Si el rango aumenta, añadimos el vector a L, si no aumenta, lo descartamos pues es
combinación lineal de los anteriores.
! !
2 1 0 2 2 1 0 2
rango = rango =2
3 4 2 1 0 5/2 2 −2

el rango aumentó, lo añadimos a la base que estamos construyendo

L = [(2, 1, 0, 2), (3, 4, 2, 1)]

38
Repetimos el proceso hasta agotar los vectores.
2 1 0 2 2 1 0 2
   

rango 0 5/2 2 −2 = rango 0 5/2 2 −2 =


   
1 3 2 −1 0 5/2 2 −2

2 1 0 2
 

rango 0 5/2 2 −2 = 2


 
0 0 0 0
El rango no aumentó, descartamos el vector (1, 3, 2, −1) pues es combinación lineal de
los anteriores.
L = [(2, 1, 0, 2), (3, 4, 2, 1)]
2 1 0 2
 

rango 0 5/2 2 −2 = 3


 
0 0 1 −1
El rango aumentó, lo añadimos a L
L = [(2, 1, 0, 2), (3, 4, 2, 1), (0, 0, 1, −1)]
Se cumple que
V = ⟨(2, 1, 0, 2), (3, 4, 2, 1), (0, 0, 1, −1)⟩
y estos vectores son independientes. Por tanto forman una base y dim(V ) = 3.
Ejemplo 5.0.9. Sea V = R5 . Tenemos que
L = [(1, 1, 0, 0, 0), (1, −1, 0, 0, 0), (0, 0, 0, 0, 2)]
es una familia libre. Calculemos una base de V que contenga a L. Como dim(V ) = 5
debems añadir dos vectores. Podrı́amos añadir vectores al azar y comprobar si son base,
pero para asegurarnos de que siempre terminamos, podemos tomar vectores de la base
canónica. Probamos con e1

1 1 0 0 0 1 1 0 0 0
   
1 −1 0 0 0 0 −2 0 0 0
rango   = rango  =
   
0 0 0 0 2 0 0 0 0 2
1 0 0 0 0 0 −1 0 0 0
1 1 0 0 0
 
0 −2 0 0 0
rango  =3
 
0 0 0 0 2
0 0 0 0 0
No nos sirve, probamos con e2
1 1 0 0 0 1 1 0 0 0
   
0 −2 0 0 0 0 −2 0 0 0
 
rango   = rango  =3
 
0 0 0 0 2 0 0 0 0 2
0 1 0 0 0 0 0 0 0 0

39
5 Bases, dimensión, Fórmula de Grassmann

Tampoco sirve. Es fácil comprobar que e3 , e4 si van a servir

1 1 0 0 0
 
0 −2 0 0 0
 
rango 0 0 0 0 2 = 5
 
0 0 1 0 0
 

0 0 0 1 0

Por lo que una base como la solicitada puede ser:

[(1, 1, 0, 0, 0), (1, −1, 0, 0, 0), (0, 0, 0, 0, 2), (0, 0, 1, 0, 0), (0, 0, 0, 1, 0)]

Teorema 5.0.10.

• Si U ⊆ V entonces dim(U ) ≤ dim(V ).

• Si U ⊆ V y dim(U ) = dim(V ) entonces U = V .

• Si U = ⟨v1 , . . . , vr ⟩ entonces dim(U ) ≤ r.

• Si dim(V ) = n y [v1 , . . . , vn ] son linealmente independientes, automáticamente es


una base de V .

• Si dim(V ) = n y [v1 , . . . , vn ] es un conjunto generador, automáticamente es una


base de V .

Definición 5.0.11.

1. dim(0) = 0, es el único espacio vectorial de dimensión cero.

2. Un espacio de dimensión 1 se llama recta.

3. Un espacio de dimensión 2 se llama plano.

4. Si V tiene dimensión n, los subespacios de V de dimensión n − 1 se llaman hiper-


planos.

Teorema 5.0.12. La intersección de subespacios vectoriales siempre es un subespacio


vectorial.

Si U y V son espacios vectoriales. U ∩ V es el espacio vectorial más grande que está


contenido tanto en U como en V .

dim(U ∩ V ) ≤ mı́n{dim(U ), dim(V )}

La intersección de dos espacios V1 , V2 se puede calcular a partir de las ecuaciones


implı́citas.

40
Ejemplo 5.0.13. El plano P de R3 dado por las ecuaciones {2x − y + 3z = 0}
y la recta R de ecuaciones {x − 3y = 0, y + 2z = 0} la intersección es:

{2x − y + 3z = 0, x − 3y = 0, y + 2z = 0}

Si resolvemos el sistema es el vector (0, 0, 0).

En general, la unión de espacios vectoriales NO es un espacio vectorial. La única


excepción es cuando uno de ellos está contenido en el otro.

2 U1 = ⟨u1 ⟩
u1
U2 = ⟨u2 ⟩ 1

u1 + u2
0
−2 −1 0 1 2 3

−1
u1
−2

−3

La unión de U1 y U2 no es un subespacio.

Definición 5.0.14. La suma de espacios vectoriales U, V es el espacio vectorial más


pequeño que contiene tanto a U como a V .

Teorema 5.0.15. Si {u1 , . . . , ur } son generadores de U y {v1 , . . . , vs } son generadores


de V , entonces
U + V = ⟨u1 , . . . , ur , v1 , . . . , vs ⟩
Ası́ máx{dim(U ), dim(V )} ≤ dim(U + V ) ≤ dim(U ) + dim(V ).

Observación 5.0.16. {u1 , . . . , ur , v1 , . . . , vs } es conjunto generador, pero no tiene por qué


ser una base.
Todos los vectores w de U + V son de la forma

w = u + v, u ∈ U, v ∈ V

U + V = {u + v | u ∈ U, v ∈ V }

Teorema 5.0.17. Sean U, W ⊆ V dos subespacios, entonces se verifica la fórmula de


Grassmann
dim(U + W ) = dim(U ) + dim(W ) − dim(U ∩ W )

41
5 Bases, dimensión, Fórmula de Grassmann

Demostración. Sea dim(U ) = n, dim(W ) = m, dim(U ∩W ) = r. Debe ser r ≤ mı́n{n, m}.


Tomemos una base de U ∩ W , BU ∩W = [v1 , . . . , vr ]. Sabemos que podemos encontrar
bases de U y W que contienen a esta familia libre de vectores:

BU = [v1 , . . . , vr , u1 , . . . , un−r ]

BW = [v1 , . . . , vr , w1 , . . . , wm−r ]
Entonces tenemos el conjunto generador de U + W

U + W = ⟨v1 , . . . , vr , u1 , . . . , un−r , v1 , . . . , vr , w1 , . . . , wm−r ⟩

eliminando vectores repetidos:

U + W = ⟨v1 , . . . , vr , u1 , . . . , un−r , w1 , . . . , wm−r ⟩

Veamos que estos vectores son independientes. Supongamos una combinación nula:

a1 v1 + · · · + ar vr + b1 u1 + · · · + bn−r un−r + c1 w1 + · · · + cm−r wm−r = 0

tenemos que comprobar que las constantes deben ser nulas. Pasando los w a la derecha

a1 v1 + · · · + ar vr + b1 u1 + · · · + bn−r un−r = −c1 w1 + · · · − cm−r wm−r

El vector p = −c1 w1 + · · · − cm−r wm−r está en W , pero p = a1 v1 + · · · + ar vr + b1 u1 +


· · · + bn−r un−r por lo que p ∈ U y p ∈ U ∩ W . Por tanto, se puede escribir en función de
la base de U ∩ W .

p = −c1 w1 + · · · − cm−r wm−r = d1 v1 + . . . + dr vr

para ciertas constantes di . Despejando

d1 v1 + . . . + dr vr + c1 w1 + · · · cm−r wm−r = 0

como [v1 , . . . , vr , w1 , . . . , wm−r ] es una familia libre, las constantes di y ci deben ser cero.
Sustituyendo en la combinación nula original.

a1 v1 + · · · + ar vr + b1 u1 + · · · + bn−r un−r + 0w1 + · · · + 0wm−r = 0

por lo que
a1 v1 + · · · + ar vr + b1 u1 + · · · + bn−r un−r = 0
de nuevo [v1 , . . . , vr , u1 , . . . , un−r ] es linealmente independiente, por lo que las constantes
ai y bi deben ser nulas. Tenemos ası́ que:

[v1 , . . . , vr , u1 , . . . , un−r , w1 , . . . , wm−r ]

son independientes y forman una base de U + W . Ası́

dim(U + V ) = r + n − r + m − r = n + m − r = dim(U ) + dim(W ) − dim(U ∩ W )

42
Ejemplo 5.0.18. Sea P1 = {2x − y + 3z = 0}, P2 = {x + y + z = 0} subespacios de R3 .
P1 y P2 son planos de R3 , dim(P1 ) = dim(P2 ) = 2.
P1 ∩ P2 = ⟨(−4, 1, 3)⟩
Una recta.
dim(P1 + P2 ) + dim(P1 ∩ P2 ) = dim(P1 ) + dim(P2 )
dim(P1 + P2 ) + 1 = 2 + 2
Sin necesidad de calcularlo, la fórmula de Grassmann me dice que P1 + P2 = R3 . En
efecto P1 = ⟨(−3, 0, 2), (1, 2, 0)⟩, P2 = ⟨(1, −1, 0), (1, 0, −1)⟩ y
{(−3, 0, 2), (1, 2, 0), (1, −1, 0)}
es base de R3 .
En R3 , si dim(P1 ) = dim(P2 ) = 2 son dos planos
dim(P1 ∩ P2 ) dim(P1 + P2 ) posible
0 4 NO
1 3 SI
2 2 SI
3 1 NO
4 0 NO
Teorema 5.0.19. En un espacio vectorial, si U y V son subespacios, son equivalentes:
• dim(U ∩ V ) = 0
• dim(U + V ) = dim(U ) + dim(V )
• La unión de una base de U y una base de V es siempre una base de U + V .
• Cada vector de U + V se escribe de forma única como un vector de U más un
vector de V .
Diremos, en este caso, que la suma es directa y escribiremos U ⊕ V en lugar de
U +V.
Ejemplo 5.0.20. Si P = {x + y + z = 0}, R = ⟨(1, 2, 3)⟩ en R3 , vemos que la intersección
es cero, luego la suma será directa.
El vector w = (−5, −5, 0) se escribirá, de manera única como w = u + v, u ∈ P , v ∈ U .
• Calculamos una base de P = ⟨(1, −1, 0), (1, 0, −1)⟩
• Calculamos una base de R = ⟨(1, 2, 3)⟩
• Como la suma es directa, (1, −1, 0), (1, 0, −1), (1, 2, 3) es base de P + R = R3
• Escribimos (−5, −5, 0) como combinación de la base.
(−5, −5, 0) = 5/3(1, −1, 0) − 5(1, 0, −1) − 5/3(1, 2, 3)
u = 5/3(1, −1, 0) − 5(1, 0, −1) = (−10/3, −5/3, 5)
v = −5/3(1, 2, 3) = (−5/3, −10/3 − 5)

43
5 Bases, dimensión, Fórmula de Grassmann

5.1. Problemas
Ejercicio 5.1.1. Calcule el rango de las siguientes matrices:

1 −1 1 −1
 

1 2 3 0 1 −1 1 
 
 
4 5 6 2 1 0 1
   
7 8 9 1 1 1 1
 

2 0 2 0

Ejercicio 5.1.2. Calcule todas las soluciones del sistema:


(
x + 0y − z = −2
3x + 0y − 3z = −6

Ejercicio 5.1.3. Calcule el determinante y la inversa de la matriz

1 −1 −2 −10
 

 −1 2 2 10 

−1 6 3 15 
 

0 −4 2 11

Ejercicio 5.1.4. ¿Es Q2 un subespacio vectorial de R2 considerados como R-espacios?


¿Y como Q-espacios?

Ejercicio 5.1.5. ¿Es la recta {(x, y)|x − y = 2} un subespacio vectorial de R2 ?

Ejercicio 5.1.6. Demuestre que si {u, v} ⊆ V es una familia libre y a es una constante
cualquiera, entonces {u, v + au} también es una familia libre.

Ejercicio 5.1.7. Sean {v1 , v2 , v3 } ⊆ V una de vectores, tales que {v1 , v2 }, {v1 , v3 } y
{v2 , v3 } son familias independientes ¿Es {v1 , v2 , v3 } una familia independiente?

Ejercicio 5.1.8. Calcule todas las soluciones en R2 de los siguientes sistemas. ¿Cuáles
de ellos definen subespacios
( vectoriales de R ?
2
(
n 2x + y = 1 3x + y = 5
2x − y = 0
x − 3y = 0 −6x − 2y = 5

Ejercicio 5.1.9. Compruebe si las siguientes familias son linealmente dependientes o


independientes.

• {(2, 0, 1), (1, 2, 0), (5, 2, 2)} en R3

• {1 + x, 2 − x2 , 1 + x + 2x2 } en R[x]<3

Ejercicio 5.1.10. Dados los siguientes subespacios U , V , W de R3 , extraiga una base


de los generadores dados. Para cada una de estas bases, extienda esa base a una de R3 .

• U = ⟨(1, 0, 1), (0, 0, 0), (1, 1, 2), (0, −1, −1), (0, 1, 1)⟩

44
5.1 Problemas

• V = ⟨(1, 2, 3), (4, 5, 6), (7, 8, 9), (10, 11, 12)⟩

• W = ⟨(0, 1, 1), (1, 0, 1), (0, 0, 0), (1, 1, 1)⟩


Ejercicio 5.1.11. Calcule las ecuaciones implı́citas de los subespacios U , V de R3 del
ejercicio anterior.
Ejercicio 5.1.12. Sabemos que en R3 toda base tiene tres elementos. ¿Significa esto
que dados tres vectores no repetidos forman una base de R3 ?
Ejercicio 5.1.13. En R4 , sean

U = ⟨(1, 0, 1, 1), (0, 1, 0, 0)⟩,

V = ⟨(0, 0, 0, 0), (0, 1, 1, 1), (0, 0, 1, 1), (1, 0, 0, 0), (0, 2, 3, 3)⟩.
Calcula la suma de U + V y la intersección U ∩ V ¿Es la suma directa? ¿Se cumple la
fórmula de las dimensiones? ¿Es cierto siempre que si U + V = V entonces U ∩ V = U ?
¿por qué?
Ejercicio 5.1.14. Sea U = ⟨(1, −1, 1), (0, 2, 3)⟩, V = ⟨(0, 1, 1), (−1, 0, 1)⟩. Para cada
vector ei de la base canónica, escribe ese vector como la suma de un vector ui ∈ U y un
vector vi ∈ V , ei = ui + vi ¿Es esa manera de escribirlo única? ¿Es U + V suma directa?
Ejercicio 5.1.15. En Z52 , calcule U ∩ V ∩ W donde:
• U = {(x, y, z, t, u)|x + y + z + t = 0}

• V = {(a, b, a + b, 0, a)|a, b ∈ Z2 }

• W = ⟨(0, 1, 1, 1, 0), (0, 1, 1, 0, 0), (0, 0, 1, 0, 1), (1, 0, 1, 1, 0), (1, 1, 0, 1, 0)⟩
Ejercicio 5.1.16. En R4 , sea V = {(x, y, z, t)|x−y+z−t = 0}, U = ⟨(1, 0, 1, 0), (0, 1, 0, 1)⟩.
Tenemos que (1, 0, 1, 0) ∈
/ V , (0, 1, 0, 1) ∈
/ V . ¿Significa esto que U ∩ V = {0}?
Ejercicio 5.1.17. Calcule
1. Un espacio V de R6 de dimensión 4.

2. Un subespacio U de V de dimensión 1.

3. Un subespacio T de R6 tal que V = U ⊕ T .

4. De las ecuaciones implı́citas y paramétricas de U, V, T .


Ejercicio 5.1.18. Sea U un subespacio de R10 de dimensión 7. Sea V un subespacio de
R10 tal que T = U ⊕ V es un hiperplano de R10 . Sea W un subespacio de R10 . Demuestre
que:
• Si W ⊆ V , entonces U ∩ W = {0}.

• Si T ⊕ W = R10 , entonces W es una recta.

45
5 Bases, dimensión, Fórmula de Grassmann

Ejercicio 5.1.19. Sea V = M2×3 (R). Sea


( ! )
a b c
U= | |a + d = 0, b − e = 0, a + e − f = 0
d e f

Calcule tres elementos independientes de U . De dos bases distintas de U . Calcule una


base de V .

Ejercicio 5.1.20. Sean U, V ⊆ R5 , si dim(U ) = 2 y dim(V ) = 3 ¿Es R5 = U + V ?


Supongamos que R5 = U + V ¿Es R5 = U ⊕ V ?

Ejercicio 5.1.21. Sea {v1 , v2 , v3 , v4 } un sistema generador de R3 . ¿Cuáles de las siguientes


afirmaciones son ciertas?

1. R3 = ⟨v1 , v2 , v3 , v4 ⟩.

2. v1 , v2 , v3 , v4 son linealmente independientes.

3. {v1 , v2 , v3 , v4 } contiene una base de R3 .

4. Si v = c1 v1 + c2 v2 + c3 v3 + c4 v4 , los coeficientes c1 , c2 , c3 , c4 son únicos.

Ejercicio 5.1.22. Sea V un espacio vectorial de dimensión 4 y sean U1 y U2 dos


subespacios vectoriales de V . ¿Cuáles de las siguientes afirmaciones son ciertas?

1. La unión de dos bases cualesquiera de U1 y U2 es una base de U1 + U2 .

2. Si V = U1 + U2 , U1 ∩ U2 = {0} y dim(U1 ) = 2 entonces dim(U2 ) = 2.

3. Si V = U1 + U2 y dim(U1 ) = 2 entonces dim(U2 ) = 2.

4. Si V = U1 + U2 entonces todo vector de v ∈ V se puede escribir como v = u1 + u2


con u1 ∈ U1 y u2 ∈ U2 .

Ejercicio 5.1.23. Sea V un espacio de dimensión 3. ¿Cuáles de las siguientes afirmaciones


son ciertas?

1. Todas las bases de V tienen 3 elementos.

2. V tiene un subconjunto libre de 2 elementos.

3. V tiene un sistema generador de 2 elementos.

4. V tiene un sistema generador de 4 elementos.

Ejercicio 5.1.24. Sea V un espacio de dimensión 5, sean A y B subespacios de V


¿Cuáles de las siguientes afirmaciones son ciertas?

1. Puede ser dim(A) = 2.

46
5.1 Problemas

2. Si dim(A) = 2 y dim(B) = 1, puede ser V = A + B.

3. Si dim(A) = 3 y dim(B) = 4, puede ser V = A + B.

4. Si dim(A ∩ B) = 0, entonces dim(A) + dim(B) = dim(A + B).

Ejercicio 5.1.25. Sea V un espacio vectorial, con subespacios U1 y U2 ¿Cuáles de las


siguientes afirmaciones son ciertas?

1. Si uniendo una base de U1 y una base de U2 tenemos una base de V , entonces


U1 ∩ U2 = {0}.

2. dim(V ) = dim(U1 ) + dim(U2 ).

3. Si dim(U1 ) + dim(U2 ) = dim(V ) y V = U1 + U2 entonces dim(U1 ∩ U2 ) puede ser 1.

4. Si dim(U1 ) + dim(U2 ) = dim(V ) − 1 y V = U1 + U2 entonces dim(U1 ∩ U2 ) puede


ser 1.

47
6 Coordenadas, cambio de coordenadas
En este capı́tulo vamos a ver que dos espacios vectoriales sobre un cuerpo cualquier
espacio vectorial sobre K de la misma dimensión son esencialmente iguales.
Una noción importante es el concepto de coordenadas de un vector respecto de una
base.
Vamos a describir de una manera un poco distinta los conceptos de sistema generador,
libre y base. Tomemos por ejemplo R4 .

A = [(1, 0, 2, 3), (−1, 2, 1, 2), (0, 1, 2, −2), (0, 0, 1, 2), (2, 3, 4, 5), (0, 0, 0, 3)]
es un conjunto generador pero no es una base. Si queremos escribir un vector (a, b, c, d)
como combinación lineal de los vectores de A
1 0 0 2 0
             
−1 a
0  2   1  0 3 0  b 
x  + y  + z  + t  + u  + v  =  
             
2  1   2  1 4 0  c 
3 2 −2 2 5 3 d
debemos resolver el sistema:
 
x
1 −1 0 0 2 0 
   
y  a

0 2 1 0 3 0 z   b 
  =  
    
2 1 2 1 4 0  t   c 

3 2 −2 2 5 3 u
 
d
v

que el conjunto A sea generador significa que, sea quien sea (a, b, c, d) el sistema es
compatible, siempre tiene solución.

C = [(0, 1, 0, 0), (0, 0, 2, 0)]


es un conjunto linealmente independiente. Esto se puede traducir en que si queremos
escribir un vector (a, b, c, d) como combinación lineal de los vectores de C

0 0
   
! a
1 0 x b
= 
   
0 2 y c
 

0 0 d
El sistema no siempre tiene solución, pues C no es generador, pero si hay solución, esta
es única. Es decir el sistema siempre es o incompatible o compatible determinado.

49
6 Coordenadas, cambio de coordenadas

Finalmente D = [(1, 0, 0, 0), (2, 1, 0, 0), (−8, 1, 2, 2), (0, 0, 0, 1)] es una base de R4 . Esto
se traduce en que el sistema
1 2 −8 0
    
x a
0 1 1 0  y   b 
   
  =  

0 0 2 0  z   c 

0 0 2 1 t d
Siempre es compatible determinado, sea quien sea (a, b, c, d), luego la solución es única.
A esta solución la llamamos coordenadas del vector (a, b, c, d) en la base D.
Definición 6.0.1. Si V es un espacio vectorial y B = [v1 , . . . , vn ] es una base de V
entonces todo vector u ∈ V se escribe, de manera única, como
u = a1 v1 + . . . + an vn
(a1 , a2 , . . . , an )B son las coordenadas de u en la base B.
u = (a1 , a2 , . . . , an )B
Si cambiamos la base, cambiamos las coordenadas, pero no cambiamos el vector.
Ejemplo 6.0.2. Sea V = C[x]<6 , una base de V es B = [1, x, x2 , x3 , x4 , x5 ]. El polinomio
f = 1 + 3x + (4 + i)x2 + x5 tiene por coordenadas en la base B
f = (1, 3, 4 + i, 0, 0, 1)B
En cambio, en la base B1 = [i, x, x2 , x3 , x4 , x5 ] el polinomio f tiene coordenadas
f = (−i, 3, 4 + i, 0, 0, 1)B1
Esto son abreviaturas de:
f = 1 · 1 + 3 · x + (4 + i) · x2 + 1 · x5 =
(−i) · i + 3 · x + (4 + i) · x2 + 1 · x5
Ejemplo 6.0.3. Sea Bc = [(1, 0), (0, 1)] la base canónica
Sea B = [(2, 3), (−1, 0)]
El vector (4, −5)B ¿Qué coordenadas tiene en la base canónica?

(4, 5)B = 4 · (2, 3) − 5 · (−1, 0) = (13, 12)Bc


El vector (4, −5)Bc ¿Qué coordenadas tiene en la base B? Tenemos que escribir el
vector como combinación de los vectores de la base B

(4, −5)Bc = a · (2, 3) + b(−1, 0)


Resolviendo el sistema:
5 22
(4, −5)Bc = − (2, 3) − (−1, 0)
3 3
5 22
 
(4, −5)Bc = − , −
3 3 B

50
Ejemplo 6.0.4. Sea V un espacio vectorial de dimensión 5 y A = [v1 , v2 , v3 , v4 , v5 ] una
base cualquiera. Siempre se cumple:
v1 = 1v1 + 0v2 + 0v3 + 0v4 + 0v5 = (1, 0, 0, 0, 0)A
v3 = 0v1 + 0v2 + 1v3 + 0v4 + 0v5 = (0, 0, 1, 0, 0)A
v4 + v5 = (0, 0, 0, 1, 1)A
Si u = (a1 , a2 , a3 , a4 , a5 )A y w = (b1 , b2 , b3 , b4 , b5 )A . Entonces:
u + w = (a1 + b1 , a2 + b2 , a3 + b3 , a4 + b4 , a5 + b5 )
λu = (λa1 , λa2 , λa3 , λa4 , λa5 )
Ejemplo 6.0.5. Tomemos R2 y las bases canónica Bc = [(1, 0), (0, 1)] y A = [(2, 0), (1, −1)].
Tomemos el vector:
v = (3, 1) = (3, 1)Bc = 2(2, 0) − (1, −1) = (2, −1)A
una representación gráfica de las coordenadas es la siguiente:
2 2

v = 3e1 + 1e2 v = 2(2, 0) − (−1, 1)


1 1
1e2 −1(1, −1)
0 0
−1 0 1 2 −1 0 1 2
3e1 2(2, 0)
−1 −1

Teorema 6.0.6. Dadas dos bases B1 = [v1 , v2 , . . . , vr ] y B2 = [w1 , . . . , wr ] de un espacio


vectorial V de dimensión r existe una única matriz MB1 B2 llamada la matriz de cambio
de base de B1 a B2 tal que. Si (a1 , a2 , . . . , ar )B1 son las coodenadas de un vector v en
la base B1 y (b1 , b2 , . . . , br )B2 son las coordenadas del mismo vector v en la base B2 , es
decir si
v = a1 v1 + a2 v2 + . . . + ar vr = b1 w1 + b2 w2 + . . . + br wr
entonces    
b1 a1
b2  a2 
   
.
. = MB1 B2 
 .. 

. .
br B2
ar B1
Demostración. Describimos cómo debe ser la matriz. La matriz se multiplica por un
vector r × 1 y devuelve un vector r × 1 como resultado, por lo que debe ser una matriz
cuadrada r × r. Si tomamos v1 = (1, 0, 0, . . . , 0)B1 tenemos que:
1
 

0
 
 .. 
MB1 B2 ·  
.
0

51
6 Coordenadas, cambio de coordenadas

Por un lado, este producto devuelve la primera columna de MB1 B2 . Por otro estas deben
ser las coordanadas de v1 en la base B2 . Esto ocurre para cada columa:
La columna i-ésima de MB1 B2 son las coordenadas de vi , el i-ésimo vector de la base
de salida respecto de la base de llegada.
Vemos que esto determina de manera única la matriz. Para ver que esta cumple lo
pedido, sea
v = (a1 , . . . , ar )B1
entonces
v = a1 v1 + . . . ar vr
Si denotamos por Ci la columna i de la matriz, las coordenadas de v en la base B2 será

a1
 
 .. 
a1 C1 + a2 C2 + . . . + ar Cr = MB1 B2 · . 
ar

Ejemplo 6.0.7. Consideremos las bases de R3 siguientes:

A = [(1, −1, 2), (2, 3, 1), (1, 0, 1)]

B = [(1, 2, 0), (2, 1, 1), (0, 1, 0)]


La matriz MAB debe ser una matriz 3 × 3.
 
a11 a12 a13
MAB = a21 a22 a23 
 
a31 a32 a33

Tomemos v1 = (1, −1, 2) = (1, 0, 0)A . Sus coordenadas en la base B son la solución de:

(1, −1, 2) = x(1, 2, 0) + y(2, 1, 1) + z(0, 1, 0)

x = −3 y = 2 z = 3
Luego la matriz MAB debe cumplir:

1
     
a11 a12 a13 −3
a21 a22 a23  · 0 =  2 
     
a31 a32 a33 0 A 3 B

Esta igualdad nos determina la primera columna:


 
−3 a12 a13
MAB =  2 a22 a23 
 
3 a32 a33

52
De manera análoga, la segunda columna viene determinada por las coordenadas de
(0, 1, 0)A en la base B

(0, 1, 0)A = (2, 3, 1) = x(1, 2, 0) + y(2, 1, 1) + z(0, 1, 0)

x=0 y=1 z=2


y la tercera columna viene determinada por las coordenadas del terer vector de la base A
en la base B
(1, 0, 1) = x(1, 2, 0) + y(2, 1, 1) + z(0, 1, 0)
x = −1 y=1 z=1
Ası́:
−3 0 −1
 

MAB = 2 1 1 
 
3 2 1
Por ejemplo:
−3 0 −1 2
     
−9
 2 1 1  · −3 =  4 
     
3 2 1 3 A 3 B
Es decir:
1 2 1 1 2 0
           

2 −1 − 3 3 + 3 0 = −9 2 + 4 1 + 3 1


           
2 1 1 0 1 0
En este ejemplo:
(2, −3, 3)A = (−9, 4, 3)B = (−1, −11, 4)

Teorema 6.0.8. Si B1 , B2 , B3 son bases de V .

MB1 B3 = MB2 B3 MB1 B2

Demostración. Sea v ∈ V , entonces:

(MB2 B3 MB1 B2 )vB1 = MB2 B3 (MB1 B2 vB1 ) = MB2 B3 vB2 = vB3

Es decir MB2 B3 MB1 B2 es una matriz que si la multiplicamos por las coordenadas de un
vector en la base B1 nos devuelve las coordenadas de dicho vector en la base B3 . Es decir,
es la matriz de cambio de base de B1 a B3

MB1 B3 = MB2 B3 MB1 B2

Cabe destacar que la definición de multiplicación de matrices se inventa para que la


anterior igualdad se cumpla.

53
6 Coordenadas, cambio de coordenadas

Ejemplo 6.0.9. Si V es un espacio vectorial de dimensión n, y A, B son bases:

1 0 ... 0
 

0 1 . . . 0
 
MAA = Idn = 

.. 
 . 
0 0 ... 1

es la matriz identidad. De aquı́ y la igualdad:

MAA = MBA MAB

tenemos que:
In = MBA MBA
es decir:
−1
MBA = MAB
El cambio de coordenadas inverso viene dado por la matriz inversa.

Ejemplo 6.0.10. En R3 , Tomemos las bases A = [(1, 2, 3), (4, 5, 6), (7, 0, 0)], B =
[(0, 1, 0), (1, 1, 1), (2, 8, 15)]. Entonces la matriz de cambio de base MABc es la matriz que
tiene por columnas las coordenadas de los vectores de A escritos en la base canónica.

1 4 7
 

MABc = 2 5 0
 
3 6 0

de manera análoga:
0 1 2
 

MBBc = 1 1 8 
 
0 1 15
Entonces:
−1 
0 1 2 1 4 7
 
−1
MAB = MBc B MABc = MBB MABc = 1 1 8  2 5 0 
   
c
0 1 15 3 6 0
−1 
1 4 7 0 1 2
 
−1 −1
MBA = MAB = MAB MBBc = 2 5 0 1 1 8 
   
c
3 6 0 0 1 15

54
6.1 Ejercicios

6.1. Ejercicios
Ejercicio 6.1.1. Calcule las matrices de cambio de base en los siguientes casos:

1. En R2 MAB con A = [(1, 0), (0, 1)], B = [(2, 3), (1, 1)].

2. En R2 MAC con A = [(1, 0), (0, 1)], C = [(5, 1), (1, 0)].

3. En R2 MBC con B = [(2, 3), (1, 1)], C = [(5, 1), (1, 0)]

Ejercicio 6.1.2. Sea A = [u, v, w] una base de un espacio V .

• ¿Qué vector tiene por coordenadas (1, 2, 3)A ? (Escrı́balo en función de los vectores
[u, v, w].

• ¿Qué coordenadas tiene el vector u − v en la base A?

• ¿Qué coordenadas tiene el vector u − v en la base [v, w, u]?

Ejercicio 6.1.3. En R3 tenemos las bases

A = [(1, 0, 1), (2, 0, 1), (1, 1, 0)], B = [(1, −1, 0), (2, 0, 1), (−3, 2, −1)].

Calcule las matrices de cambio de base MAB y MBA .

Ejercicio 6.1.4. Sea T = [u, v, w] una base de un espacio V . Demuestre que S =


[u, u + v, w + u] también es una base de V . ¿Cuales son las matrices de cambio de base
MT S y MST ?

Ejercicio 6.1.5. Sea V = P({1, 2, 3, 4, 5}) el conjunto de todos los subconjuntos de


{1, 2, 3, 4, 5}. Es decir z ∈ V si y solo si z ⊆ {1, 2, 3, 4, 5}. Vamos a dotar de estructura
de espacio vectorial a V sobre el cuerpo Z/(2).
Para el producto por un escalar:

0 · x = ∅, 1·x=x

Para la suma de dos elementos de V :

x +V y = (x\y) ∪ (y\x)

En esta estructura la suma es el conjunto que contiene a los elementos que están en uno
y solo uno de los conjuntos x, y.
Por ejemplo:
{1, 2, 3} +V {2, 4, 5} = {1, 3, 4, 5}
{2, 3} +V {2, 3} = ∅
Con estas operaciones V es un espacio vectorial.

• Demuestre que el vector cero es 0V = ∅.

55
6 Coordenadas, cambio de coordenadas

• Demuestre que B = [{1}, {2}, {3}, {4}, {5}] es una base de V . ¿Qué dimensión tiene
V?

• ¿Cuales son las coordenadas en la base B de los vectores ∅, {1, 5}, {2, 4}?

• Compruebe que la suma de las coordenadas de {1, 2, 3} y {2, 4, 5} son las coordenadas
de {1, 3, 4, 5}.
Ejercicio 6.1.6. En R3 tenemos el plano dado en implı́citas por

U = {(x, y, z) | x + y + z = 0}.

Una base de U es A = [(0, 1, −1), (1, 1, −2)].


• Si tomamos el vector v = (2, −3, 1) ∈ U ¿Cuales son las coordenadas de v en la base
A?

• ¿Por qué v solo tiene dos coordenadas en la base A?

• ¿Tiene sentido preguntar las coordenadas de (1, 0, 0) ∈ R3 en la base A?

• Dado un vector cualquiera w = (x, y, z) ∈ U dado por sus coordenadas en la base


canónica de R3 ¿Cuales son las coordenadas de w en la base U ?
Ejercicio 6.1.7. En (Z/(2))2 tenemos solamente una cantidad finita de bases:
• Bc = [(1, 0), (0, 1)]

• B2 = [(1, 0), (1, 1)]

• B3 = [(0, 1), (1, 0)]

• B4 = [(0, 1), (1, 1)]

• B5 = [(1, 1), (1, 0)]

• B6 = [(1, 1), (0, 1)]


Como tenemos seis posibilidades como base de salida y seis posibilidades de la base de
llegada. En total existen 36 matrices de cambio de base en el plano. Calcúlelas (Nota, solo
existen seis matrices regulares distintas de tamaño dos, por lo que cada matriz aparecerá
seis veces repetida en el listado anterior).
Ejercicio 6.1.8. En R3 tenemos la matriz

0 2 3 2
   
−1 −1
M =  1 0 −1  M −1 = 2 3 −3 
   
1 1 1 −1 −2 2

1. Calcule una base A de R3 para que la matriz de cambio de base de la canónica a A


sea la matriz M .

56
6.1 Ejercicios

2. Calcule una base B de R3 para que la matriz de cambio de base de B a la base


canónica sea la matriz M .

3. Dada la base C = [(1, 0, 1), (0, 1, 2), (0, 0, 1)] calcule una base D para que la matriz
de cambio de base de C a D sea la matriz M .

Ejercicio 6.1.9. Sea V un espacio vectorial de dimensión 4. A = [v1 , v2 , v3 , v4 ], B =


[w1 , w2 , w3 , w4 ] dos bases de V . Si

1
 
∗ ∗ ∗
∗ ∗ 0 ∗
MAB =
 
∗ ∗ 0 ∗

∗ ∗ 0 ∗

quiere decir que v3 = w1 . ¿Cual es la base B en el siguiente caso?

0 0 1 0
 
0 1 0 0
MAB =
 
0 0 0 1

1 0 0 0

Calcule la matriz MAB si B = [v2 , v3 , v4 , v1 ]


Si v = (2, 3, −1, 5)A ¿Cuales son las coordenadas de v en la base T = [v4 , v3 , v1 , v2 ]?
Estas matrices que tienen solo un uno en cada fila y en cada columna se llamam
matrices de permutación. Como matrices de cambio de baase solo reordenan los vectores
de la base.

57
7 Aplicaciones lineales
En este capı́tulo estudiamos funciones entre distintos espacios vectoriales. Muchas
de estas funciones tienen un sabor geométrico, giros, simetrı́as, proyecciones etc. Sin
empargo no es completamente geométrico porque tenemos la restricción de que el vector
cero es especial.

Definición 7.0.1. Si V y W son espacios vectoriales sobre el mismo cuerpo K. Una


aplicación lineal
f :V →W

Es una aplicación que conserva las combinaciones lineales. Es decir, si u, v ∈ V y a, b ∈ K


entonces
f (au + bv) = af (u) + bf (v)

Ejemplo 7.0.2. La aplicación

f: R3 → R4
(x, y, z) 7→ (2x, −y, z, 0)

es lineal.
En efecto, sean v = (a, b, c) y w = (d, e, g) dos vectores y α, β dos constantes.

αv + βw = (αa + βd, αb + βe, αc + βg)

f (αv + βw) = (2(αa + βd), −(αb + βe), αc + βg, 0) =

= α(2a, −b, c, 0) + β(2d, −e, g, 0) =

= αf (a, b, c) + βf (d, e, g)

Teorema 7.0.3. Sean f : V → W , g : W → U dos aplicaciones lineales.

• f (0V ) = 0W

• f (−v) = −f (v)

• Si {v1 , . . . , vk } es una familia dependiente de V , entonces la familia de W {f (v1 ), . . . , f (vk )}


es dependiente.

• g ◦ f : V → U también es lineal.

59
7 Aplicaciones lineales

Ejemplo 7.0.4. La aplicación

f : R −→ R
x 7→ x + 1

No es lineal, pues no lleva el cero al cero.

f (0) = 1 ̸= 0

La aplicación
ℜ: C → C
z 7→ ℜ(z)
Que lleva a cada número complejo sobre su parte real f (a + b · i) = a es lineal sobre R
pero no sobre C.

Ejemplo 7.0.5. Bit de paridad


A cada byte
(x1 , x2 , x3 , x4 , x5 , x6 , x7 , x8 )
le asignamos un 1 si el número de unos es impar y cero si el número de unos es par.

p: Z82 → Z2
u = (x1 , x2 , x3 , x4 , x5 , x6 , x7 , x8 ) 7→ p(u)

Es una aplicación lineal sobre Z2 . En realidad

p(u) = x1 + x2 + x3 + x4 + x5 + x6 + x7 + x8

Ejemplo 7.0.6. Si V = U ⊕ W , es suma directa, entonces cada vector v ∈ W se expresa


de manera única como v = uv + wv , con uv ∈ U , wv ∈ W . La aplicación

π: V → U
v = uv + wv 7→ uv

Es lineal.

π(a(uv1 + wv1 ) + b(uv2 + wv2 )) = auv1 + buv2


aπ(uv1 + wv1 ) + bπ(uv2 + wv2 ) = auv1 + buv2
La proyección de V sobre U en la dirección de W .

Ejemplo 7.0.7. R4 = ⟨(2, 3, −1, 2), (1, 4, 5, 0)⟩ ⊕ ⟨(2, −2, 2, 1), (3, 0, 4, 1)⟩
v1 = (2, 3, −1, 2), v2 = (1, 4, 5, 0), v3 = (2, −2, 2, 1), v4 = (3, 0, 4, 1).
Si v ∈ R4 , existen únicos a, b, c, d con

v = av1 + bv2 + cv3 + dv4

π(v) = av1 + bv2 ∈ ⟨v1 , v2 ⟩

60
R2 = ⟨(1, −1)⟩ ⊕ ⟨(1, 1)⟩
π, la proyección sobre ⟨(1, −1)⟩ en la dirección ⟨(1, 1)⟩.

a−b a+b
(a, b) ∈ R2 = (1, −1) + (1, 1)
2 2

π: R2 → ⟨(1, −1)⟩
(a, b) 7→ a−b b−a
2 , 2

⟨(1, 1)⟩

⟨(1, −1)⟩

Ejemplo 7.0.8. Si V tiene dimensión n y W tiene dimensión m. Para cualquier matriz


M m × n la aplicación
V → W
v 7→ M · v
es lineal. (Siempre entenderemos v como vector columna).

Ejemplo 7.0.9. Consideremos el cuerpo K = Z23 . La parte numérica del DNI se puede
interpretar como vector en K8 . Como cada cifra del DNI está solamente entre 0 y 9, el
conjunto de todos los DNI NO es un subespacio de K8 .
Por otro lado, las letras del DNI posibles son

Let = {T RW AGM Y F P DXBN JZSQV HLCKE}


Vamos a dar una estructura de espacio vectorial a las letras Let del DNI, identificando
cada letra con un elemento de K:

0 1 2 3 4 5 6 7 8 9 10 11
T R W A G M Y F P D X B
12 13 14 15 16 17 18 19 20 21 22
N J Z S Q V H L C K E

Hay una aplicación lineal de K8 en Let que hace corresponder a cada DNI su le-
tra. El DNI 00000001 se corresponde con la tupla (0, 0, 0, 0, 0, 0, 0, 1). El 12345678 con
(1, 2, 3, 4, 5, 6, 7, 8) etc. Pero hay tuplas que no se corresponden con ningún DNI como
(13, 8, 20, 18, 2, 3, 11, 8).

61
7 Aplicaciones lineales

(a1 , a2 , a3 , a4 , a5 , a6 , a7 , a8 ) 7→
7→ a1 Z + a2 Y + a3 L + a4 H + a5 B + a6 P + a7 X + a8 R
Esta forma de calcular el DNI como aplicación lineal es correcta, pero no suele exponerse
de esta forma. Busque cómo se presenta el cálculo de letra del DNI y compárelo con lo
anterior.
Definición 7.0.10. Si f : V → W es lineal. El Núcleo de f , escrito como ker(f ) es el
conjunto de vectores de V que van a parar al cero de W
ker(f ) = {v ∈ V | f (v) = 0W }
Si v1 , v2 ∈ ker(f ) y a, b ∈ K, entonces
f (av1 + bv2 ) = af (v1 ) + bf (v2 ) = a0W + b0W
Por tanto ker(f ) es un subespacio de V .
Ejemplo 7.0.11.
g: R3 → R2

x ! !
1 −1 0 x−y
(x, y, z) 7→ · y  =
 
0 1 −1 y−z
z

ker(g) = {(x, y, z) ∈ R3 |(x − y, y − z) = (0, 0)} =


= {x − y = 0, y − z = 0}
En este caso ker(g) = ⟨(1, 1, 1)⟩
Definición 7.0.12. Si f : V → W es lineal. La Imagen de f , escrito como Im(f ) es el
conjunto de vectores de W que son imagen de algún vector de V .
Im(f ) = {f (v) ∈ W | v ∈ V }
Si w1 , w2 ∈ Im(f ) y a, b ∈ K, entonces existe v1 ∈ V , v2 ∈ V con f (v1 ) = w1 , f (v2 ) = w2 .
aw1 + bw2 = f (av1 + bv2 ) ∈ Im(f )
Por tanto Im(f ) es un subespacio vectorial de W .
Ejemplo 7.0.13.
g: R3 → R2

! x !
1 −1 0 x−y
(x, y, z) 7→ · y  =
 
0 1 −1 y−z
z

Im(g) = {f (x, y, z)|(x, y, z) ∈ R3 } =


= {(x − y, y − z)|x, y, z ∈ R}
En este caso Im(g) = ⟨(1, 0), (−1, 1), (0, −1)⟩ = R2

62
Definición 7.0.14. f : V → W lineal
Una aplicación lineal es inyectiva si ker(f ) = {0V }. En este caso, si f (v) = f (w) entonces
v = w.
Una aplicación lineal es sobreyectiva si Im(f ) = W . En este caso, todo elemento de W
proviene de algún elemento de V .
Una aplicación es biyectiva si es a la vez inyectiva y sobreyectiva.
Teorema 7.0.15. Si f : V → W es lineal:
dim(V ) = dim(ker(f )) + dim(Im(f ))
Además
• Si f es inyectiva dim(V ) ≤ dim(W ).
• Si f es sobreyectiva dim(V ) ≥ dim(W ).
• Si f es biyectiva dim(V ) = dim(W ).
• Si f es biyectiva f −1 existe y es lineal.
Sea f : V → W una aplicación lineal y sean B = [v1 , . . . , vn ], B ′ = [w1 , . . . , wm ] bases
de V y W . Tenemos entonces que
f (vi ) = a1i w1 + . . . + ami wi , 1 ≤ i ≤ n
Si u ∈ V , u = b1 v1 + . . . + bn vn , sus coordenadas en B son
u = (b1 , . . . , bn )B
f (u) = b1 f (v1 ) + . . . + bn f (vn ) ¿Cómo son las coordenadas de la imagen en B ′ ?
 
a11 a12 . . . a1n b1
 
a a22 . . . a2n   . 

 ·  .. 
 21
f (u)B ′ = 
 ... 
am1 am2 . . . amn bn

Teorema 7.0.16. Dada una aplicación lineal f : V → W y un par de bases B de V


y B ′ de W , existe una única matriz MB,B ′ (f ) tal que, si u = (x1 , . . . , xn )B y f (u) =
(y1 , . . . , ym )B ′ , entonces    
y1 x1
 y2   x2 
   
 .  = MB,B ′ (f )  . 
 .   . 
 .   . 
ym xn

V = ⟨v1 , . . . , vn ⟩ W = ⟨w1 , . . . , wm ⟩
 
a11 a12 . . . a1n b1
 
a a22 . . . a2n   . 

 ·  .. 
 21
f (u)B ′ = 
 ... 
am1 am2 . . . amn bn B
La columna p de la matriz son las coordenadas de f (vp ) en la base B ′ .

63
7 Aplicaciones lineales

Ejemplo 7.0.17.
ϕ : R[x]<4 → R[x]<3
f 7→ f (3) + f ′ (0) + f ′′
Tomamos las bases canónicas ⟨1, x, x2 , x3 ⟩, ⟨1, x, x2 ⟩

• ϕ(1) = 1(3) + 1′ (0) + 1′′ = 1 = (1, 0, 0)B ′

• ϕ(x) = x(3) + 1(0) + 0 = 4 = (4, 0, 0)B ′

• ϕ(x2 ) = x2 (3) + 2x(0) + 2 = 11 = (11, 0, 0)B ′

• ϕ(x3 ) = x3 (3) + 3x2 (0) + 6x = 27 + 6x = (27, 6, 0)B ′

f = a + bx + cx2 + dx3 = (a, b, c, d)B


   
a a
1 4 11 27
 
b b
ϕ   = 0 0 0 6  ·  
     
c c
0 0 0 0
d B d B

Sea f : V → W una aplicación lineal. Sean A, B bases de V y C, D bases de W .


Sea M la matriz coordenada de f cuando tomamos la base A en V y la base C en W .
Sea N la matriz coordenada de f cuando tomamos la base B en V y la base D en W .

¿Qué relación hay entre M y N ?

uA f (u)C
f
VA WC
M

MBA MCD

f
VB WD
N
uB f (u)D

N · uB = MCD · M · MBA · uB

Definición 7.0.18. Dos matrices M y N son equivalentes si están asociadas a la


misma aplicación lineal en distintas bases. Es decir, si existen matrices regulares tales
que M = QN P .

64
Teorema 7.0.19. Sea f : V → W una aplicación lineal n = dim(V ), m = dim(W ),
r = dim(Im(f )). Entonces existen bases A = [v1 , . . . , vn ] de V y B = [w1 , . . . , wm ] de W
tales que: !
Ir 0
MAB (f ) =
0 0
Es la matriz m × n que tiene r 1 en la diagonal y el resto de elementos son cero.

Demostración. Veamos que condiciones deben cumplir las bases para que la matriz
coordenada tenga esta estructura. Como las columnas de la matriz codifican las imágenes
de la base A, se tiene que:

f (vi ) = (0, . . . , 0, 1i) , 0, . . . , 0)B = wi si 1 ≤ i ≤ r

f (vi ) = 0W si r < i
Las codiciones que deben verificar A y B son:

1. dim(ker(f )) = n − r, los últimos n − r vectores de A deben ser una base del núcleo.

2. Los r primeros vectores de B son las imágenes de los r primeros vectores de A.

De esta manera, A y B no son únicas. Siempre podemos encontrar una base de A con
los últimos n − r vectores base del núcleo con el método de amplicar la base.
Si A = [v1 , . . . , vr , vr+1 , . . . , vn ] es esa base, sus imágenes son

[f (v1 ), . . . , f (vr ), 0W , . . . , 0W ]

Como este conjunto genera Im(f ) y dim(Im(f )) = r, tenemos que f (v1 ), . . . , f (vr ) son
independientes en W y, por tanto podemos completar a una base.

Ejemplo 7.0.20. Sea f : R4 → R3 que, en bases canónicas, tiene por matriz coordenada:

1 0 1 2
 

MBc Bc (f ) =  −1 −3 2 −2 
 
1 2 −1 2

Esta matriz tiene rango 2, por lo que dim(Im(f )) = 2 y, por tanto, dim(ker(f )) = 4 − 2.
Buscamos bases T y S para que:

1 0 0 0
 

MT S = 0 1 0 0
 
0 0 0 0

Si T = [v1 , v2 , v3 , v4 ] y S = [w1 , w2 , w3 ] son bases que nos dan la matriz buscada, debe
cumplirse que:

f (v1 ) = w1 , f (v2 ) = w2 , f (v3 ) = 0R3 , f (v4 ) = 0R3

65
7 Aplicaciones lineales

El núcleo de la función son las soluciones de


 
x
1 0 1 2   0
  
 y   
−1 −3 2 −2 = 0

z 
   
1 2 −1 2 0
t
Calculando:
ker(f ) = ⟨(1, 1, 1, −1), (0, 2, 2, −1)⟩
por lo que podemos tomar v3 = (1, 1, 1, −1) y v4 = (0, 2, 2, −1). Completamos estos
vectores con v1 y v2 para tener una base de R4 . Por ejemplo:
T = [(0, 0, 1, 0), (0, 0, 0, 1), (1, 1, 1, −1), (0, 2, 2, −1)]
Sea quien sea la base S la matriz coordenada tiene sus dos últimas columnas cero.
∗ ∗ 0 0
 

MT S = ∗ ∗ 0 0
 
∗ ∗ 0 0
Ahora, para que la primera columna sea (1, 0, 0) debe ser f (v1 ) = (1, 0, 0)S = w1 , por
lo que w1 = f (v1 ) = (1, 2, −1). DE la misma manera f (v2 ) = (0, 1, 0)S = w2 , por lo que
w2 = f (v2 ) = (2, −2, 2). Finalmente añadimos un vector w3 para obtener una base de R3 .
Por ejemplo w3 = (1, 0, 0).
S = [(1, 2, −1), (2, −2, 2), (1, 0, 0)]
De manera matricial
MT S (f ) = MBc S (Id)MBc Bc (f )MT Bc (Id)
0 0 1 0
 
−1 
1 0 0 0 1 2 1 1 0 1 2 
   
 0 0 1 2
0 1 0 0 =  2 −2 0  −1 −3 2 −2  
     
1 0 1 2

0 0 0 0 −1 2 0 1 2 −1 2
0 1 −1 −1
Teorema 7.0.21. Sea M una matriz n × m de rango r, entonces M es equivalente a la
matriz !
Ir 0
0 0
Teorema 7.0.22. Dos matrices son equivalentes si y solo si tienen el mismo tamaño y
el mismo rango.
Demostración. Sean M, N dos matrices de tamaño m × n y rango r. Interpretándolas
como funciones lineales. Hay matrices de cambio de base tales que:
!
Ir 0
QM P = = SN T
0 0
despejando:
(S −1 Q)M (P T −1 ) = N

66
7.1 Problemas

7.1. Problemas
Ejercicio 7.1.1. Dadas las siguientes matrices, calcule el espacio de salida, el espacio de
llegada, una base del núcleo, otra de la imagen y verifique que se cumplen la fórmula de
las dimensiones.
Sobre R:
1 1 2 3
   
!
0 0 0  
, 1 2 3 , 2 , 4 5 6
   
0 0 0
3 7 8 9
3 −3 0
 
1 −1 −2
 
!
 6 −3 −3 1 0 −1
0 1 −1  , ,
   
 1 0 −1 2 2 −2

1 −1 −10
−2 1 1
Sobre Z2 :
0 1 0 0 
 
0 1 0 0 0
  
0 0 0 1 

 , 1 1  , 0 1 0 
   
0 0 0 1

1 0 0 0 0
0 1 0 1
Ejercicio 7.1.2. Se da la matriz de una aplicación lineal f entre dos espacios vectoriales
sobre R, V y W en las bases canónicas. También se da una base B1 de V y una base B2
de W . Se pide calcular la matriz de la aplicación f respecto de las bases B1 y B2 .
!
1 1
1. M = ,
0 1
B1 = [(2, 0), (1, −1)], B2 = [(1, 1), (1, −3)].

2 −47 −102
 

2. M = 29 −32 55 ,
 
3 −18 −29
B1 = [(−23, −12, 5), (68, 35, −15), (−4, −2, −1)],
B2 = [(4, −4, 1), (7, 9, 3), (−16, 3, −5)]
 
3. M = 1 0 0 1 ,
B1 = [(1, 0, 1, 0), (0, 1, 0, 1), (1, 0, −1, 0), (0, 0, 0, 1)], B2 = [1/3].
!
1 0
4. M = ,
0 −1
B1 = [(1, 0), (0, 1)], B2 = [(2, 3), (3, 5)].
Ejercicio 7.1.3. De un ejemplo de aplicaciones lineales f, g : R3 → R3 tal que ker(f ) ⊆
Im(f ) y Im(g) ⊆ ker(g). ¿Podemos tener una aplicación lineal h : R3 → R3 con
ker(h) = Im(h)? ¿y si consideramos h : R4 → R4 ?
Ejercicio 7.1.4. ¿Es posible encontrar matrices A ∈ M3×2 (R) y B ∈ M2×3 (R) tal que
AB tenga rango 3?

67
7 Aplicaciones lineales

Ejercicio 7.1.5. Sean U, V, W espacios vectoriales. Sean B1 , B2 bases de U , B3 , B4


bases de V y B5 , B6 bases de W . Sea f : U → V y g : V → W aplicaciones lineales. Si
A = MB1 ,B3 (f ), C = MB4 ,B5 (g), describa la matriz de la aplicación MB2 ,B6 (g ◦ f ) como
producto de A, C y matrices de cambio de base.
Ejercicio 7.1.6. Dadas las siguientes matrices M , encuentra matrices P y Q tales que
P M Q es diagonal con unos y ceros en la diagonal.
! !
1 1   0 0
1 2 −3
1 1 0 0

0
 
−28 −7 −14
1 2 0 
 
 −18 2 88 −1 
−1 −2 0 
 
 0 8 −3 0 

2 4 0
−1 −4 −1 −6
Ejercicio 7.1.7. Sea V un espacio vectorial de dimensión 3, u, v, w una base de V y
f : V → V tal que:
f (v) = u + w f (u) = u ker(f ) = ⟨v + w⟩
Calcule Im(f ), ker(f 2 ), ker(f 3 ).
Ejercicio 7.1.8. Sea f : R3 → R2 una aplicación lineal. Consideramos BV = {e1 , e2 } la
base canónica de R2 y BW = {e1 , e2 , e3 } la base canónica de R3 ¿Cuáles de las siguientes
afirmaciones son ciertas?
1. La matriz coordenada de f con respecto a BW y BV , MBW ,BV (f ), tiene tres filas y
dos columnas.
2. La matriz coordenada de f con respecto a BW y BV , MBW ,BV (f ), tiene dos filas y
tres columnas.
3. Las columnas de la matriz MBW ,BV (f ) son un sistema generador de Im(f ).
4. dim(ker(f )) ̸= 0.
Ejercicio 7.1.9. Determine si las siguientes afirmaciones son verdaderas o falsas:
1. Si V = ⟨v1 , v2 , v3 ⟩ y f (v1 ) = f (v3 ), entonces dim(Im(f )) ≤ 2.
2. Si v1 ∈ ker(f ) y v2 ∈
/ ker(f ), entonces v1 + v2 ∈ ker(f ).
3. Si f : R3 → R2 es sobreyectiva, entonces dim(ker(f )) = 2.
4. Si f : R3 → R4 , cumple que ker(f ) es un plano, entonces la imagen de f es una
recta.
5. Si A = [(1, 1), (1, 0)] y B = [(2, 1), (−1, 2)] son bases. Entonces
(1, 0)A = (1, 1)B .
6. Si A = [(1, 1), (1, 0)] y B = [(2, 1), (−1, 2)] son bases. Entonces
(1, 0)B = (1, 1)A .

68
8 Teorı́a del Endomorfismo
Comenzamos con un ejemplo de la quı́mica:

Ejemplo 8.0.1. Reacciones reversibles:


En presencia de un catalizador, tenemos la siguiente reacción quı́mica reversible

2SO2 + O2 ⇌ 2SO3

En condiciones ideales, cada unidad de tiempo, el 40 % de SO3 se descompone y el 16 %


de SO2 reacciona con el oxı́geno para formar SO3 . En un matraz tenemos disuelto:

• 64 gramos de SO2 (1 mol de SO2 )

• 128 gramos de O2 (4 mol de O2 )

• 800 gramos de SO3 (10 mol de SO3 )

Si dejamos el sistema evolucionar, ¿Qué ocurrirá con él? En cada unidad de tiempo
(expresado en mols):

xSO2 (t) → 0,84xSO2 (t + 1) + 0,16xSO3 (t + 1)


zSO3 (t) → 0,6zSO3 (t + 1) + 0,2zO2 (t + 1) + 0,4zSO2 (t + 1)
yO2 (t) → yO2 (t + 1) − x0,08SO2 (t + 1)

Es decir
x(t + 1) 0,84 0,0 0,4
     
x(t)
y(t + 1) =  −0,08 1 0,2  · y(t)
     
z(t + 1) 0,16 0,0 0,6 z(t)
En n unidades de tiempo
n 
x(t + n) 0,84 0,0 0,4
   
x(t)
y(t + n) =  −0,08 1 0,2  · y(t)
     
z(t + n) 0,16 0,0 0,6 z(t)

0,84 0,0 0,4 0,00 1,0 −0,75


   

Sea A =  −0,08 0,92 0,2 , P =  1,0 −0,00024 −0,38 


   
0,16 0,0 0,6 0,00 0,38 0,75

1,0 −2,1 × 10−17 2,1 × 10−17 1,00 0 0


   
−1
P AP ∼  0,00 1,0 1,4 × 10−17
∼ 0 1,00 0 
   
0,00 −2,8 × 10−17 0,44 0 0 0,44

69
8 Teorı́a del Endomorfismo

Tenemos además que:

(P −1 AP )n = (P −1 AP )(P −1 AP ) · · · (P −1 AP )(P −1 AP ) =

= P −1 A(P P −1 )A(P P −1 ) · · · A(P P −1 )AP =


P −1 AA · · · AP = P −1 An P
En vez de evaluar cómo evoluciona el sistema calculando potencias de A, podemos calcular
potencias de P −1 AP
An = P (P −1 AP )n P −1
1n 0 0
 
−1
(P AP ) ∼  0 1n
n
0 
 
0 0 0,44n
El lı́mite cuando tiene n a infinito es
1 0 0
 

0 1 0
 
0 0 0

An = P (P −1 AP )n P −1 tiende a

1 0 0 0,71 0,0 0,71


   

P 0 1 0 P −1 =  −0,14 1,0 0,35 


   
0 0 0 0,28 0,0 0,28

La concentración inicial era 1 mol SO2 , 4 mol de O2 , 10 mol SO3 .


La concentración final será
0,71 0,0 0,71 1 7,85
     

−0,14 1,0 0,35 · 4 = 7,42


     
     
0,28 0,0 0,28 10 3,14

1 ∗ 64 + 4 ∗ 32 + 80 ∗ 10 = 992 7,85 ∗ 64 + 7,42 ∗ 32 + 3,14 ∗ 80 = 991,04 La diferencia de


masa viene de los errores de redondeo, una milésima, del mismo orden que la precisión
de la matriz.
Una concentración (x, y, z) está en equilibrio en este modelo si
   
x x
A · y  = y 
   
z z

Llamaremos a estos vectores autovectores asociados al autovalor uno.


Los puntos de equilibrio son la variedad lineal ⟨(1, 0, 0,4), (0, 1, 0)⟩, nos está diciendo que
el oxı́geno es irrelevante para estar en equilibrio (lo que no es cierto, si no hay oxı́geno
no puede darse una de las reacciones, hay soluciones matemáticas que no son soluciones
fı́sicas).

70
Por otro lado, como 0.44 está en la diagonal, hay un vector tal que
   
x x
A · y  = 0,44 y 
   
z z
Estos vectores forman el espacio ⟨(2, 1, −2)⟩ que no tiene interpretación fı́sica evidente.
Sin embargo, sugiere que puede tener que ver con las constantes del ajuste de la reacción.
2SO2 + 1O2 ⇌ 2SO3
Desde el punto de vista del álgebra lineal, estamos trabajando en transformaciones de
un espacio en si mismo.
f :V →V
Ejemplo 8.0.2. Trabajemos en R2 , el giro de π/6 viene dado por la matriz
√ 
3
2 − 12
G=
 


1 3
2 2

La simetrı́a de eje de simetrı́a ⟨(1, 0)⟩ y dirección de simetrı́a ⟨(1, 1)⟩ viene dado por la
matriz !
1 −2
S=
0 −1
Ambas matrices son de rango 2, luego son congruentes. Existe matrices P y Q de
cambio de base tales que:
S = P GQ
Sin embargo esto no es satisfactorio en este caso, pues solo estamos trabajando con las
dimensiones del núcleo e imagen de ambas aplicaciones. Una respuesta más fina para
distinguirlos es buscar puntos de equilibrio. Por ejemplo, si buscamos vectores (x, y) que
verifiquen: √ 
3 1
− ! !
 2 2
 x x

=
y y
 
1 3
2 2
solo tenemos la solución trivial (0, 0) puesto que un vector no nulo, si lo giramos π/6
radianes siempre obtenemos otro vector.
Sin embargo para la simetrı́a:
! ! !
1 −2 x x
=
0 −1 y y
Tiene por solución precisamente el eje de simetrı́a ⟨(1, 1)⟩. Una simetrı́a tiene autovectores
asociados al autovalor 1 (el eje de simetrı́a). Pero un giro de π/6 no tiene ninguno.
La clave en este caso es que, como tanto el espacio de salida como el de llegada es el
mismo, para dar un significado más profundo a la aplicación lineal, debemos trabajar
con la misma base en ambos espacios.

71
8 Teorı́a del Endomorfismo

A lo largo de este capı́tulo siempre trabajamos en la misma base en el espacio de salida


y de llegada.

Definición 8.0.3. Llamamos endomorfismo a una aplicación lineal de un espacio en si


mismo:
f :V →V
Sea f : V → V un endomorfismo. Un autovector o vector propio de f es un vector
v ̸= 0, tal que existe una constante α ∈ K de manera que:

f (v) = αv

Al valor α se le llama valor propio o autovalor asociado.

Si v es un vector propio de f asociado al valor propio α. Sea g = f − αId, tenemos


que g(v) = f (v) − αv = 0. Por tanto v ∈ ker(g) y el núcleo es no trivial.
Es decir,

Teorema 8.0.4. Los valores propios de f son los valores α tales que la función lineal
f − αIdV tiene núcleo distinto de cero.

Definición 8.0.5. Sea M una matriz cuadrada, el polinomio caracterı́stico de M es


el determinante de
x ... 0
 

.


 ..  − M


0 ... x

Teorema 8.0.6. Si f un endomorfismo y M es un matriz asociada en una base B,


entonces α es un valor propio de f si y solo si α es raı́z del polinomio caracterı́stico de f .

Ejemplo 8.0.7. Consideremos la función lineal f : R2 → R2 que, en bases canónicas


tiene matriz coordenada: !
1 2
A=
2 1
Su polinomio caracterı́stico es:

x − 1 −2
|xI2 − A| = = (x − 1)2 − 4 = x2 − 2x − 3

−2 x − 1

una constante t es raı́z del polinomio caracterı́stico si y solo si el sistema de ecuaciones


lineales ! ! ! !
1 2 x x tx
=t =
2 1 y y ty

Tiene solución (x, y) ̸= (0, 0). Es decir si hay un vector propio asociado a t. Las raı́ces de
x2 − 2x − 3 son 3 y −1. Por tanto la función f tiene vectores propios asociados a 3 y −1

72
y no tiene más vectores propios. Para el valor propio 3, (x, y) es vector propio asociado a
3 si se cumple: ! ! !
1 2 x 3x
=
2 1 y 3y
Es decir, si ! ! !
1−3 2 x 0
=
2 1−3 y 0
Es decir, (x, y) ∈ ker(f − 3Id) = ⟨(1, 1)⟩. Los vectores propios asociados a 3 son todos
los vectores de la recta ⟨(1, 1)⟩ (excepto el (0, 0)). Para el valor propio −1, calculamos el
núcleo de: ! !
1 − (−1) 2 2 2
=
2 1 − (−1) 2 2
que es la recta ⟨(1, −1)⟩.
! ! ! !
1 2 1 3 1
= =3
2 1 1 3 1
! ! ! !
1 2 1 −1 1
= = (−1)
2 1 −1 1 −1

Definición 8.0.8. Sean M , N , dos matrices de orden n. Diremos que M y N son


matrices semejantes si existe una matriz regular P con P −1 M P = N . Es decir, si son
matrices coordenadas de un mismo endomorfismo respecto de dos bases distintas.

Dos matrices semejantes siempre son congruentes, pero el recı́proco no es cierto en


general.

Teorema 8.0.9. Sea f un endomorfismo. Entonces el polinomio caracterı́stico no depende


de la base escogida.

Demostración. Si M es la matriz de f es una base A (tanto en el espacio de salida como


el de llegada) y N es la matriz de f en la base B y P es la matriz de cambio de base de
A a B. Entonces:
M = P −1 N P
|xId − M | = |xId − P −1 N P | = |xP −1 P − P −1 N P |
por la propiedad distributiva y asociativa de las operaciones con matrices

|xP −1 P − P −1 N P | = |P −1 (xId − N )P |

Por las propiedades de los determinantes:

|P −1 (xId − N )P | = |P |−1 |xId − N ||P | = |xId − N |.

El polinomio caracterı́stico, y los valores propios, son los mismos en ambas bases.

73
8 Teorı́a del Endomorfismo

Ejemplo 8.0.10. En el Ejemplo 8.0.7, la función f , en bases canónicas, tiene por matriz
!
1 2
A=
2 1

Tomemos una base de R2 , T = ⟨(1, 1), (1, 0)⟩.


!
1 1
MT Bc =
1 0
!−1 !
1 1 0 1
MBc T = =
1 0 1 −1
La matriz coordenada de f en la base T es:
! ! ! !
0 1 1 2 1 1 3 2
MT T (f ) = MBc T AMT Bc = =
1 −1 2 1 1 0 0 −1

El polinomio caracterı́stico de MT T es

x − 3 −2
= (x − 3)(x + 1) − 0 = x2 − 2x − 3

−0 x − (−1)

que es el mismo que el de la matriz A.

De esta manera, como el polinomio caracterı́stico no depende de ninguna base, podemos


hablar del polinomio caracterı́stico de la función f .

Teorema 8.0.11. Sea f : V → V un endomorfismo y xn + an−1 xn−1 + . . . + a1 x + a0 su


polinomio caracterı́stico. Sea M la matriz coordenada de f respecto de cualquier base.
Entonces:

• −an−1 es la traza de M (la suma de los términos de la diagonal).

• (−1)n a0 es el determinante de M .

La cuestión esencial que queremos responder es determinar si una matriz es semejante


a una matriz diagonal. Es decir, dado un endormorfismo decidir si existe una base en la
cual la matriz asociada sea diagonal.

Definición 8.0.12. Sea M una matriz cuadrada. Diremos que M es diagonalizable


si es semejante a una matriz diagonal. Es decir si existe una matriz P regular tal que
P −1 M P es diagonal.

Teorema 8.0.13. Sea f un endomorfismo, f es diagonalizable si y solo si existe una


base V formada por vectores propios. En ese caso, la matriz diagonal semejante tiene los
valores propios en la diagonal.

74
Demostración. Basta observar que en la base A = [v1 , . . . , vn ] que la matriz MAA (f )
tengamos en la columna i-ésima el vector (0, . . . , 0, αi) , 0, . . . , 0) significa que f (vi ) = αvi
es decir que vi sea vector propio asociado al valor propio αi .
!
1 2
Ejemplo 8.0.14. Siguiendo con el Ejemplo 8.0.7. A = MBc Bc (f ) = . Sus valores
2 1
propios son 3 y −1 y tenemos la base de vectores propios ⟨(1, 1), (1, −1)⟩. Por ser vectores
propios:
f (1, 1) = 3(1, 1) = (3, 3)
f (1, −1) = −1(1, −1) = (−1, 1)
En la base S = [(1, 1), (1, −1)], la matriz coodenada de f es:
 
1 1 ! ! !
2 2
 1 2 1 1 3 0
=

2 1 1 −1 0 −1
 
1 −1
2 2

Luego A es diagonalizable y es semejante a la matriz diagonal que contiene sus valores


propios.
!
0 −1
Ejemplo 8.0.15. Sea A = de un giro de π/4 radianes en R2 .
1 0

Gπ/4 : R2 → R2 .

Geométricamente, ningún vector (distinto del vector 0) despues de girarlo queda


proporcional a si mismo. Eso quiere decir que no vamos a tener vectores propios.
Si calculamos el polinomio caracterı́stico, obtenemos el polinomio x2 + 1 que no tiene
raı́ces reales. Por tanto esta matriz no es diagonalizable.
Si embargo si consideramos la misma función con números complejos:

Gπ/4 : C2 → C2 .

Ahora el polinomio caracterı́stico tiene por valores propios i, −i, con vectores propios:
! ! ! !
0 −1 i −1 i
= =i
1 0 1 i 1
! ! ! !
0 −1 −i −1 −i
= = −i
1 0 1 −i 1
Tenemos la base de C2 formada por vectores propios [(i, 1), (−i, 1)]. En esta base la
matriz del giro es diagonal. !
i 0
0 −i
Por lo que esta matriz no es diagonal en R2 pero si en C2 .

75
8 Teorı́a del Endomorfismo
!
0 1
Ejemplo 8.0.16. Sea K cualquier cuerpo y A = . El polinomio caracterı́stico
0 0
de A es x2 , luego su único valor propio es α = 0. Pero al buscar los vectores propios
asociados a cero solamente obtenemos ker(A) = ⟨(1, 0)⟩. No existe ninguna base de K2
formada por vectores propios. Por tanto A no es diagonalizable.

Definición 8.0.17. Sea f un endormorfismo y α un valor propio. Llamamos espacio


propio asociado a α y lo denotamos por Vα a ker(f − αId). Nótese que Vα (salvo el
vector 0) es el conjunto de todos los vectores propios asociados a α.

Al buscar una base de vectores propios, podrı́a ocurrir que al mezclar vectores propios
asociados a valores propios distintos, estos fueran dependientes. Veamos que no es el caso.
Siempre van a ser, automáticamente independientes.

Teorema 8.0.18. Sea M una matriz cuadrada y α1 , . . . , αr valores propios distintos de


M (no necesariamente todos). Para cada i sea:

Bi = [vi1 , . . . , visi ] una base de Vαi

Entonces
B = [v11 , . . . , v1s1 , v21 , . . . , v2s2 , . . . , vr,sr ]
es una familia independiente.

Demostración. Lo demostramos por inducción en r el número de valores propios. La


propiedad es cierta para r = 1, pues en este caso B = B1 que es libre por ser base de V1 .
Supongamos cierto el resultado para r = k y probémoslo para k + 1. Si B = B1 ∪ . . . ∪
Bk ∪ Bk+1 es una familia dependiente, tendremos una combinación lineal no trivial:

a11 v11 + . . . + a1s1 v1s1 + a21 v21 + . . . + aksk vksk + ak+1,1 vk+1,1 + . . . ak+1,s vk+1,s = 0

Agrupamos los vectores según el espacio propio en el que se encuentran luego tendremos:

u1 + u2 + . . . + uk + uk+1 = 0

donde:

• cada ui ∈ Vi .

• no puede ser uk+1 = 0, pues si lo fuera tendrı́amos que B1 ∪. . .∪Bk serı́a dependiente
al contrario que la hipótesis de inducción. No pueden ser u1 = u2 = . . . = uk = 0
pues entonces Bk+1 no serı́a base.

Entonces:
u1 + u2 + . . . + uk = −uk+1
Multiplicamos por la matriz M , por estar ui ∈ Vi , se cumple que M ui = αi ui . Ası́

α1 u1 + α2 u2 + . . . αk uk = αk+1 uk+1 = αk+1 ui + αk+1 u2 + . . . αk+1 uk

76
restando:
(α1 − αk+1 )u1 + (α2 − αk+1 )u2 + . . . (αk − αk+1 )uk = 0
como alguno de estos vectores es distinto de cero, debe ser αi − αk+1 = 0 para algún
1 ≤ i ≤ k. Esto contradice las hipótesis. Por tanto, si los valores propios son distintos. B
debe ser independiente. Por inducción, esto es cierto para cualquier número de valores
propios distintos.

Teorema 8.0.19. Sea f un endomorfismo de V , donde dim(V ) = n y sean α1 , . . . , αr


sus valores propios. Sea c(x) = (x − α1 )t1 · · · (x − αr )tr el polinomio caracterı́stico, por lo
que cada raı́z αi tiene una multiplicidad ti Entonces:

• 1 ≤ dim(Vα1 ) ≤ ti

• f es diagonalizable si y solo si i dim(Vαi ) = n.


P

• Si todas las raı́ces del polinomio caracterı́stico de f son simples, entonces f es


diagonalizable.

Ejemplo 8.0.20.
!
1 −2
• es diagonalizable como endomorfismo de C2 , pero no sobre R2 .
1 1
!
1 1
• no es diagonalizable.
0 1

Ejemplo 8.0.21. Sea f : R5 → R5 la función lineal que tiene matriz coordenada en


bases canónicas:
2 18 −144 −108
 
−37

 −156 −195 1298 −4740 −2334 

A= −205 −205 1413 −5254 −2627 
 
−108 −88 625 −2365 −1201 
 

121 81 −594 2290 1182

Su polinomio caracterı́stico es:


x + 37 144 108

−2 −18
156 x + 195 −1298 4740 2334

205 205 x − 1413 5254 2627 =

108 88 −625 x + 2365 1201

594 −2290 x − 1182



−121 −81

= x5 + 2x4 − 10x3 − 8x2 + 33x − 18 = (x − 2)(x − 1)2 (x + 3)2


Los valores propios son

• 2 con multiplicidad 1

77
8 Teorı́a del Endomorfismo

• 1 con multiplicidad 2

• −3 con multiplicidad 2

Calculemos los espacios propios asociados: Para el espacio propio V2 , tenemos que x = 2
es un valor propio simple, por lo que dim(V2 ) = 1.

2 18 −144 −108
 
−39

 −156 −197 1298 −4740 −2334 

V2 = ker(A − 2I5 ) = ker  −205 −205 1411 −5254 −2627  =
 
−108 −88 625 −2367 −1201 
 

121 81 −594 2290 1180

1
 
 24/7 
 
⟨ 9/2 ⟩
 
 17/7 
 

−39/14
que es una recta como suponı́amos.
Para el valor propio x = 1 es una raı́z doble, por lo que 1 ≤ dim(V1 ) ≤ 2.

333 0
   
 0   333 
   
V1 = ker(A − I5 ) = ⟨ 333  ,  333 ⟩
   
 322   137 
   

−491 −121

Para V1 obtenemos la máxima dimensión posible dado el polinomio caracterı́stico. Final-


mente 1 ≤ dim(V−3 ) ≤ 2 y la matriz será diagonalizable si y solo si dim(V−3 ) = 2:

2
 
−2
 
V−3 = ker(A − (−3)I5 ) = ⟨ 0 ⟩
 
 1 
 

−2

dim(V2 ) + dim(V1 ) + dim(V−3 ) = 4 < 5 por lo que la matriz no es diagonalizable.


Sin embargo, si tomamos una base que contenga a los vectores propios conseguiremos
que sea diagonal en cuatro columnas.
Tomamos una nueva base que contenga a los vectores propios por ejemplo:

24 9 17 39
 
[(333, 0, 333, 322, −491) , (0, 333, 333, 137, −121) , 1, , , ,− ,
7 2 7 14
1
 
1, −1, 0, , −1 , (1, 0, 0, 0, 0)]
2

78
8.1 El polinomio mı́nimo

Por el Teorem 8.0.18 los cuatro primeros vectores son automáticamente independientes,
pues los hemos obtenido concatenando bases de V1 , V2 y V−3 . Para el último escogemos
un vector (de la base canónica) independiente de los anteriores.
La matriz de cambio de base es:

333 0 1 1 1
 

 0 333 24
7 −1 0 

P = 333 333 9
0 0
 
2 
322 137 17
2 0
1
 
7
 
−491 −121 − 14 −1 0
39

y
1 0 0 0 172
 
333

 0 1 0 0 568 
333 
−1
P AP =  0 0 2 0 −210 
 
0 0 0 −3 4 
 

0 0 0 0 −3
que no es diagonal en la quinta columna porque el quinto vector de la base escogida
no es vector propio.
Veamos algunos ejemplos de aplicaciones vectoriales en Rn y cómo clasificarlas según
sus valores propios
Ejemplo 8.0.22. Giros en R2 : Un!giro de α grados tiene por matriz coordenada en
cos(α) − sin(α)
bases canónicas: . No es un endormorfismo diagonalizable sobre R
sin(α) cos(α)
(salvo los casos triviales α = π, 2π). Su polinomio caracterı́stico es:
c(x) = x2 − 2 cos(α)x + 1
Sus valores propios complejos son cos(α) ± i sin(α). Su determinante es 1 y su inversa
(giro de −α grados) viene dado por la matriz transpuesta.
Ejemplo 8.0.23. Proyecciones: Una proyección es un endomorfismo diagonalizable f
que cumple que f 2 = f . Son los endomorfismos diagonalizables con únicos autovalores 0
y 1. V0 es el espacio de proyección y V1 la dirección de proyección.
Ejemplo 8.0.24. Simetrı́as: Una simetrı́a es un endomorfismo diagonalizable f que
cumple que f 2 = Id. Son endomorfismos diagonalizables con únicos autovalores 1 y −1.
V1 es el eje de simetrı́a y V−1 la dirección de proyección.

8.1. El polinomio mı́nimo


Definición 8.1.1. Sea A ∈ Mn×n (K) una matriz n × n con coeficientes en K y sea
f = an xn +an−1 xn−1 +· · ·+a1 x+a0 ∈ K[x] un polinomio en la variable x con coeficientes
en K. Sea In la matriz identidad n × n. Llamamos f (A) a la matriz:
f (A) = an An + an−1 An−1 + · · · + a1 A + a0 In

79
8 Teorı́a del Endomorfismo

Diremos que el polinomio f anula a la matriz A si f (A) = (0) es la matriz cero.


!
1 0
Ejemplo 8.1.2. Sea A = . Sea f = 2x − 3 y g = x2 − 2x + 1. Entonces:
1 1
! ! !
1 0 1 0 −1 0
f (A) = 2 −3 =
1 1 0 1 2 −1
f no anula a A.
!2 ! ! !
1 0 1 0 1 0 0 0
g(A) = −2 + =
1 1 1 1 0 1 0 0
g anula a A.
Ejemplo 8.1.3. Una simetrı́a es una función lineal f tal que f ◦ f = Id. Si A es la
matriz de la simetrı́a, significa que A2 = In . Por tanto, si A es la matriz de una simetrı́a,
x2 − 1 anula a A.
Teorema 8.1.4. Sea A una matriz n × n. Existe un único polinomio f (x) ∈ K[x] que
cumple lo siguiente:
1. f es mónico, el coeficiente del término de mayor grado es 1.
2. f anula a A, f (A) = (0).
3. Si g anula a A, entonces g es un múltiplo de f .
Definición 8.1.5. El polinomio del teorema anterior se denomina el polinomio mı́nimo
de A.
No damos en este curso herramientas para calcular el polinomio mı́nimo de cualquier
matriz. Pero sabemos lo siguiente:
Teorema 8.1.6 (De Cayley-Hamilton). El polinomio caracterı́stico de una matriz A,
f (x) = |xIn − A| anula a la matriz A y, por tanto es un múltiplo del polinomio carac-
terı́stico.
De hecho, podemos afinar un poco más el teorema anterior. Si A es una matriz n × n
y tiene polinomio caracterı́stico f (x) y valores propios α1 , . . . , αr . Entonces el polinomio
caracterı́stico es de la forma:

f (x) = (x − α1 )m1 · · · (x − αr )mr


donde los exponentes indican la múltiplicidad de cada raı́z (simple, doble, triple, . . .).
Entonces el polinomio mı́nimo es de la forma:
g(x) = (x − α1 )t1 · · · (x − αr )tr
con 1 ≤ ti ≤ mi . El polinomio caracterı́stico y el mı́nimo tienen las mismas raı́ces, pero
la multiplicidad de cada raı́z es a lo sumo la indicada en el polinomio caracterı́stico.
Por ejemplo, si el polinomio caracterı́stico es f (x) = x2 (x − 1)(x − 2)(x + 4)2 . Tendemos
que el polinomio mı́nimo debe ser uno de los siguientes:

80
8.2 Problemas

• x(x − 1)(x − 2)(x + 4)

• x(x − 1)(x − 2)(x + 4)2

• x2 (x − 1)(x − 2)(x + 4)

• x2 (x − 1)(x − 2)(x + 4)2

Teorema 8.1.7. Si dos matrices son semejantes, entonces tienen el mismo polinomio
mı́nimo (el recı́proco no es cierto en general).
Una matriz cuadrada es diagonalizable si y solo si todas las raı́ces de su polinomio
mı́nimo son simples.

En el ejemplo anterior, si el polinomio caracterı́stico es f (x) = x2 (x − 1)(x − 2)(x + 4)2 ,


la matriz será diagonalizable si y solo si el polinomio mı́nimo es x(x − 1)(x − 2)(x + 4).

8.2. Problemas
Ejercicio 8.2.1. Sea f un endormorfismo. Sean u un vector propio asociado a α y
v un vector propio asociado a β con α ̸= β. Demuestre que u y v son linealmente
independientes.

Ejercicio 8.2.2. Sea f un endomorfismo y v, w vectores linealmente independientes.


Supongamos que f (v) = w, f (w) = v. Demuestre que v + w y v − w son vectores propios
de f .

Ejercicio 8.2.3. Calcule los autovalores y autovectores de las siguiente matrices ¿Cuales
son diagonalizables?
! ! ! !
−26 49 −15 24 2 1 −1 1
, , ,
−16 30 −8 13 −4 −2 1 1

1 −4 −2 1 −1 0 0 1 −1
     

2 −5 4  , 2 −2 0 , 5 −4 4 
     
2 −4 6 1 −1 0 6 −6 6
Calcule una forma diagonal y una base en la que sea diagonal.

Ejercicio 8.2.4. De las siguientes transformaciones en el plano, determina si son giros,


simetrı́as o proyecciones. De los giros calcula el ángulo. De las simetrı́as calcula el eje de
simetrı́a. De las proyecciones la recta de proyección y la dirección de proyección.
! ! ! !
0 1 5 20 5/13 12/13 4 −3
, , , ,
1 0 −1 −4 −12/13 5/13 4 −3
! ! √ √ !
−309 682 −7 −12 1
2 √ 2 − 1
2 √2
, ,
−140 309 4 7 1
2 2 1
2 2

81
8 Teorı́a del Endomorfismo

Ejercicio 8.2.5. Sea f : R3 → R3 un endormorfismo tal que:


• Su polinomio caracterı́stico es x(x − 1)(x − 3).
• ker(f ) = (0, 1, 1).
• (1, 0, 1) es un autovalor de f .
• f (1, 0, 1) ̸= (3, 0, 3).
• f (0, 1, 0) = (2, 1, 0).
Calcule la matriz coordenada de f en las base canónica.
Ejercicio 8.2.6. Sea f : R3 → R3 un endomorfismo tal que:
• f 2 es la aplicación nula.
• f (0, 1, 0) = (1, 1, 1)
• (0, 0, 1) ∈ ker(f )
Calcule la matriz coordenada de f en las bases canónicas. Ası́ como el polinomio carac-
terı́stico de f .¿Es f diagonalizable?

8.3. PageRank
Vamos a ver una base teórica del algoritmo que usa(?) Google para ordenar los
resultados y decidir qué páginas web son más importantes que otras respecto a una
búsqueda. Lo que hace Google en realidad es mucho más complicado y está modificado
por más factores que los simples enlaces. http://en.wikipedia.org/wiki/PageRank El
método que se presenta estudia un proceso estocástico similar al ejemplo de las hormigas
en el cubo visto en clase.
Un usuario (hormiga) está navegando por internet (cubo) ¿en qué páginas es más
probable que acabe?
El método se fundamenta en la idea de que una página web A es relevante si hay
muchas otras páginas relevantes que la enlazan. Si tenemos una serie de páginas web
x1 , . . . , xn cada una con importancia (peso) pi , queremos que:

• Cada peso sea positivo.


• Si xj está enlazada desde las páginas xi1 , . . . , xir , entonces es peso de xj es propor-
cional a la suma de los pesos de xik . Es decir:

pj = λ(pi1 + . . . + pik )
Naturalmente λ > 0.
Construyamos una matriz M , llamada matriz de adyacencia. El elemento mij en la
fila i columna j de la matriz M es un uno si la página xj tiene un enlace a la página xi .
En otro caso mij es cero. Es decir:

82
8.3 PageRank

• La fila i tiene unos en las páginas con enlaces hacia la página xi .

• La columna j tiene unos en las páginas con enlaces desde la página xj .

Si multiplicamos M por el hipotético vector de pesos, tendrı́amos:


   
p1 r1
p  r 
M ·  2 =  2
   
. . . . . .
pn rn

Donde

ri = mi1 p1 + mi2 p2 + · + min pn = pa1 + · · · + paj

donde xa1 , . . . , xaj son las páginas que enlazan a la página xi , pero esto es:

ri = mi1 p1 + mi2 p2 + · + min pn = pa1 + · · · + paj = λpi

Ası́
   
p1 p1
p  p 
M ·  2 = λ 2
   
. . . . . . 
pn pn

El vector de pesos deberı́a ser un autovector de la matriz de adyacencia. Ahora bien


¿Qué autovalor y autovector escoger?

Ejemplo 8.3.1. Supongamos que tenemos una red con ocho páginas web que están
enlazadas como se representa en el siguiente diagrama

5 6

1 8 2 3 4

Ası́, por ejemplo, la página 2 enlaza a las páginas 3 y 7 es enlazada desde las páginas
5, 8 y 3. La matriz de adyacencia es en este caso.

83
8 Teorı́a del Endomorfismo

0 0 0 0 0 0 1 0
 
0 0 1 0 1 0 0 1
 
0 1 0 1 0 0 0 0
 
0 0 1 0 0 0 0 0
 
M =
1 0 0 0 0 1 0 0

 
0 0 0 0 1 0 0 0


0 1 0 0 0 0 0 0
 

1 0 0 0 0 0 0 0

Camino aleatorio
Vamos a reinterpretar una modificación de la matriz como un camino aleatorio. Una
araña web (webbot, bot, etc.) es un programa que analiza páginas web en una red.
Vamos a suponer una vez que la araña ha analizado una página x, irá a una página y
tomada al azar entre las páginas enlazadas desde la página x. Por simplicidad, vamos
a suponer que todas las páginas tienen la misma probabilidad. Si una página web es
un camino sin salida, la araña no se mueve. Dejamos a la araña analizar la red durante
mucho tiempo.
Después de un tiempo prudencial, analizamos los datos que ha recogido la araña. Para
cada página x, podemos tratar de predecir cual es la probabilidad pi de que la araña esté
en la página x. En términos de matrices, podemos construir una matriz de transición
que controla el movimiento de la araña, esta es la matriz de incidencia M anterior,
dividiendo cada columna por una constante de manera que cada columna sume uno. El
hecho de que en cada columna todo elemento sea igual significa que a cada paso la araña
se mueve dando la misma probabilidad a cada enlace. Ahora, el término mij representa
la probabilidad de que la araña se mueva desde la página j hasta la i.

0 0 0 0 0 0 1 0
 
0 0 2 0 2 0 0 1
1 1
 
0 1
0 1 0 0 0 0
 
2
0 0 2 0 0 0 0 0
 1 
M = 1
0 0 0 0 1 0 0
 
2 
0 0 0 0 2 0 0 0
 1

0 1
0 0 0 0 0 0
 
2
1
2 0 0 0 0 0 0 0
La matriz M tiene la propiedad de que todos sus elementos son no negativos y la suma
de cada columna es uno. Estas matrices se llaman matrices estocásticas.
Se tiene los siguientes resultado.

Teorema 8.3.2 (Teorema de Frobenius). Sea M una matriz con entradas positivas,
Mij > 0. Es decir, una matriz con todos sus términos positivos. Supongamos además que
todas las columnas suman 1. Entonces:

• λ = 1 es autovalor simple de M .

84
8.3 PageRank

• Todos los autovalores λ ̸= 1 de M cumplen que |λ| < 1.

• Existe un único autovector asociado a 1 tal que ninguna de sus coordenadas es


negativa y sus coordenadas suman 1.

El problema es que las matrices de transición pueden tener términos nulos. En este
caso:

Teorema 8.3.3 (Teorema de Frobenius, matrices no negativas). Sea M una matriz de


transición. Es decir, una matriz con todos sus términos no negativos Mij ≥ 0 y tal que
las columnas suman 1. Entonces:

• λ = 1 es un autovalor de M .

• Todos los autovalores λ de M cumplen que |λ| ≤ 1.

• Existe un autovector (no único) asociado al valor propio 1 que cumple que ninguna
de sus coordenadas es negativa y tal que además sus coordenadas suman 1.

Siguiendo el ejemplo de la reacción quı́mica reversible visto en clase, si solo hay un


autovector asociado a 1, entonces este determina el comportamiento asintótico de la
operación M n · v.
Los pesos del método de PageRank para la red vienen dado por un autovector aso-
ciado a 1 con ningún coeficiente negativo y tal que la suma de los coeficientes es 1.

Este autovector codifica la probabilidad de que un bot realizando un camino aleatorio


en la red termine en una determinada página.

Matriz de transición con salto aleatorio


El problema de las matrices no negativas es que puede haber varios autovectores
asociados a 1, por lo que para la aplicación al PageRank no son útiles.
Para resolver este problema vamos a modificar la matriz de transición para que ninguna
entrada de la matriz sea cero. Vamos a considerar la siguiente modificación.
El bot se encuentra en una página web i. Para moverse realizamos lo siguiente:

• En un 90 % de los casos, se comporta como el bot anterior, se mueve a una página j


que esté enlazada desde i. Todas las páginas j enlazadas desde i tienen la misma
probabilidad.

• En el 10 % de los casos restantes, el bot se mueve a una nueva página completamente


al azar.

¿Cómo queda la matriz de transición de salto aleatorio? Tomamos la matriz de


transición normal, la multiplicamos por 0,9 y luego le sumamos la matriz que tiene todas
sus entradas igual a 0,1/N , donde N es el número total de páginas. En el Ejemplo 1,
tenı́amos la matriz de transición:

85
8 Teorı́a del Endomorfismo

0 0 0 0 0 0 1 0
 
0 0 2 0 2 0 0 1
1 1
 
0 1
0 1 0 0 0 0
 
2
0 0 2 0 0 0 0 0
 1 
M = 1
0 0 0 0 1 0 0
 
2 
0 0 0 0 2 0 0 0
 1

0 1
0 0 0 0 0 0
 
2
1
2 0 0 0 0 0 0 0
En este caso tenemos 8 nodos, por lo que multiplicamos M por 0,9 y le sumamos 0,1/8
a cada término de la matriz:

M1 = 0,9M + (0,1/8) =
0,0125 0,0125 0,0125 0,0125 0,0125 0,0125 0,913 0,0125
 

 0,0125 0,0125 0,462 0,0125 0,462 0,0125 0,0125 0,913 

0,0125 0,462 0,0125 0,913 0,0125 0,0125 0,0125 0,0125
 
 
0,0125 0,0125 0,462 0,0125 0,0125 0,0125 0,0125 0,0125
 
 
0,462 0,0125 0,0125 0,0125 0,0125 0,913 0,0125 0,0125
 
 
 

 0,0125 0,0125 0,0125 0,0125 0,462 0,0125 0,0125 0,0125 

0,0125 0,462 0,0125 0,0125 0,0125 0,0125 0,0125 0,0125
 
 
0,462 0,0125 0,0125 0,0125 0,0125 0,0125 0,0125 0,0125
En el caso de la matriz de transición sin salto aleatorio, si un bot estaba en el nodo 4,
solo podı́a moverse al nodo 3. En la matriz con salto aleatorio, si un bot está en el nodo
4, en el siguiente paso tendrá una probabilidad de moverse al nodo 3 de 91.3 %, y tiene
una probabilidad de 1.25 % de moverse a cualquier otro nodo (incluido el propio nodo 4).
Por el Teorema de Frobenius, el único autovector asociado a 1 de M1 , con todos los
términos positivos y cuyos coeficientes sumen uno codifica la probabilidad de que un bot
con salto aleatorio termine en una determinada página.
El resultado es distinto, en el caso de la matriz M sus autovalores son aproximadamente:

1, 0,5, −0,5, −1, −0,7, 0,7, −0,7i, 0,7i


Hay dos autovalores de módulo 1, {1, −1}. El autovector asociado a 1 (cuyas cifras
sumen 1) es único y vale aproximadamente:

(0,111, 0,222, 0,222, 0,111, 0,111, 0,0555, 0,111, 0,0555)

Luego los nodos de mayor peso son el 2 y el 3.


Sin embargo, para la matriz M1 en el que el bot puede saltar de cualquier nodo a
cualquier nodo, los autovalores son, aproximadamente:

1, 0,45, −0,45, −0,9, −0,63, 0,63, −0,63i, 0,63i


El único autovalor de módulo 1 es 1. El autovector asociado a 1 es:

86
8.3 PageRank

(0,111, 0,216, 0,203, 0,104, 0,124, 0,0684, 0,110, 0,0626)


donde solo hay un nodo de mayor peso, el 2. Hay una ligera probabilidad mayor de
terminar en el nodo 2 que en el nodo 3.

8.3.1. Ejercicios
Para el siguiente ejercicio vamos a usar el siguiente resultado:

Teorema 8.3.4. Una matriz A y su traspuesta son semejantes. Existe P regular con
At = P −1 AP . En particular A y At tienen el mismo polinomio caracterı́stico y los mismos
autovalores.

Ejercicio 8.3.5. Calcule la matriz de transición T del siguiente diagrama representando


una red con 10 páginas web.

Multipliquen la matriz traspuesta T t de T por el vector (1, . . . , 1) ¿Qué obtienen? Usen


el teorema para deducir que 1 es un autovalor de T .

87
8 Teorı́a del Endomorfismo

Ejercicio 8.3.6. Calcule la matriz de transición con salto aleatorio T1 de la red anterior.

Ejercicio 8.3.7. Calculen los autovalores de T1 . ¿Cuántos son reales? ¿Cuántos son
complejos? ¿Cuál es el autovalor de módulo más grande? 1

Ejercicio 8.3.8. Calculen el espacio de autovectores de T1 asociados al autovalor 1.


Calcule el único autovector asociado a uno cuyos coeficientes sumen 1. En nuestra red
¿Qué nodo tiene más peso en nuestro PageRank?

Ejercicio 8.3.9. Tomen una aproximación numérica de T1 , en Sage puedes obtenerla


multiplicando T1 por 1,0. Tomen 100 arañas y distribúyanlas en los nodos como quieran.
(Tomen un vector de 10 elementos no negativos que sumen 100). Apliquen la matriz de
transición numérica sobre este vector 100 veces. ¿Qué resultado obtienen? ¿Qué relación
tiene con el autovector calculado en el Ejercicio 8.3.8? ¿Cuanto suma la nueva distribución
de arañas? ¿Por qué no sale 100?

Ejercicio 8.3.10. En realidad, no todos los enlaces son igualmente importantes en una
página web. No es lo mismo un enlace escrito con letra grande, en negrita, resaltado en
mitad de la página que un enlace minúsculo a pie de página. ¿Hay alguna manera de
reflejar este hecho en la matriz de transición? ¿Cómo podemos introducir el hecho de que
un enlace sea más importante que otro?

8.4. Números enteros


8.4.1. La necesidad de Z
En general, si en N planteamos las ecuaciones

a+x=b a·x=b

esto es, si a y b representan números naturales y buscamos un valor x ∈ N que satisfaga


una u otra ecuación, no siempre es posible encontrarlo. Basta considerar, por ejemplo,
cualquiera de las dos situaciones siguientes.

9+x=7 7·x=9

Esas ecuaciones no tienen solución en N. Si a es un número natural cualquiera, la ecuación

x+a=0

tiene solución en N sólo cuando a = 0. Los números enteros se forman al añadir a los
números naturales (que llamaremos números enteros positivos) aquellos que surgen como
las soluciones de estas ecuaciones (que llamaremos números enteros negativos): −1 es la
solución de la ecuación x + 1 = 0; −2 es la solución de la ecuación x + 2 = 0; −3 es la
1

el módulo de un número real es su valor absoluto, el módulo de un número complejo a + bi es a2 + b2 .

88
8.4 Números enteros

solución de la ecuación x + 3 = 0; etc. Ası́ pues, nuestro nuevo conjunto puede describirse
como
Z = {. . . . . . , −3, −2, −1, 0, 1, 2, 3, . . . . . .} = {. . . , −3, −2, −1} ∪ N
y cuyos elementos también se ordenan de menor a mayor, extendiendo el orden que se
tiene en N:
. . . . . . . . < −3 < −2 < −1 < 0 < 1 < 2 < 3 < . . . . . . . .
En Z encontramos las mismas operaciones que en N, con la diferencia de que ahora la
resta siempre está definida. Si ahora se plantea en Z la ecuación

9 + x = 7,

entonces sı́ es posible encontrar un valor x ∈ Z que satisfaga dicha ecuación:

9 + x = 7 → (−9) + (9 + x) = (−9) + 7 →
→ ((−9) + 9) + x = (−9) + 7 →
→ 0 + x = (−9) + 7 →
→ x = (−9) + 7 = −2

La ultima igualdad de la expresión anterior queda justificada por las reglas que rigen la
operación suma de números enteros. Esa misma igualdad da idea de que la resta de dos
números enteros se reduce, entonces, a la suma de dos números enteros:

a − b = a + (−b)

Se entiende que es innecesario asimismo recordar las reglas que definen el producto de
enteros, pero encontramos relevante analizar el porqué de la igualdad

(−1)2 = 1

La siguiente cadena muestra cómo justificar tal afirmación:

0 = ((−1) + 1)2 =
= ((−1) + 1)((−1) + 1) =
= (−1)2 + 2 · 1 · (−1) + 12 =
= (−1)2 + (−1).

Tanto del orden como de las operaciones volveremos a hablar un poco más adelante.

8.4.2. Operaciones en Z. Estructura de (Z, +, ·)


Como en el caso de las operaciones en N, el objetivo primordial de este apartado es
enumerar las propiedades de las operaciones en Z y no la formalización de las mismas.
Respecto de la suma:

1. Asociativa: ∀a, b, c ∈ Z, (a + b) + c = a + (b + c)

89
8 Teorı́a del Endomorfismo

2. Existencia de elemento neutro, el 0: ∀a ∈ Z, a + 0 = 0 + a = a

3. Cada elemento tiene simétrico: ∀a ∈ Z, ∃b ∈ Z tal que a + b = b + a = 0. En ese


caso es costumbre decir que b es el opuesto de a y habitualmente se escribe b = −a.

4. Conmutativa: ∀a, b ∈ Z, a + b = b + a

Por esto, se dice que (Z, +) es grupo conmutativo Una consecuencia de la estructura de
(Z, +) es precisamente que:

La ecuación x + a = b siempre tiene solución en Z si a, b ∈ Z

pues para encontrar dicha solución basta sumar a la derecha de cada miembro de la
ecuación −a.
Respecto del producto:

1. Asociativa: ∀a, b, c ∈ Z, (a · b) · c = a · (b · c)

2. Existencia de elemento neutro, el 1: ∀a ∈ Z, a · 1 = 1 · a = a

3. Conmutativa: ∀a, b ∈ Z, a · b = b · a

• Además se tiene la propiedad distributiva del producto respecto de la suma:

∀a, b, c ∈ Z, ·(b + c) = a · b + a · c

Por el hecho de tener el conjunto de propiedades enumeradas anteriormente (respecto de


la suma, del producto y la distributiva del segundo respecto de la primera) se dice que

(Z, +, ·) tiene estructura de anillo conmutativo.

Concepto acerca del cual se volverá al final de este capı́tulo.

8.4.3. El orden ≤ en Z
La construcción del conjunto Z permite identificar una parte de él con el conjunto de
los números naturales. Como ya se ha dicho más arriba, dicha parte recibe el nombre de
subconjunto de los números enteros positivos, denotado a veces por Z+ ; el resto el de los
negativos, Z− A partir de esa identificación, se define en Z la relación:

a ≤ b ↔ ∃c ∈ Z+ : a + c = b,

que posee las siguientes propiedades:

1. Reflexiva: a ≤ a

2. Antisimétrica: a ≤ b ∧ b ≤ a → a = b

3. Transitiva: a ≤ b ∧ b ≤ c → a ≤ c

90
8.4 Números enteros

Esas tres propiedades definen a ≤ como una relación de orden en Z. Pero además se
verifica una cuarta propiedad que convierte a la relación ≤ en orden total:

∀a, b ∈ Z, a ≤ b ∨ b ≤ a

Este orden es compatible con las operaciones aritméticas:

Si a, b ∈ Z y a ≤ b, entonces se verifica

∀c ∈ Z, a + c ≤ b + c

∀c ∈ Z+ , a · c ≤ b · c

Ejercicio 8.4.1. ¿La relación ≤ en Z verifica la siguiente propiedad?

En cualquier familia (no vacı́a) de elementos de Z existe un entero que es


menor o igual a los restantes de dicha familia.

Conocida la respuesta anterior, indica la opción correcta de las siguientes (ver sección
relativa a N para recordar el significado de buen orden):

“La relación ≤ en Z □ SÍ □ NO es un buen orden”

8.4.4. Divisibilidad de Z
Recordamos los resultados básicos de la divisibilidad en Z.

Definición 8.4.2. Sean a, b ∈ Z dos números enteros. Diremos que a divide a b si


existe un entero c con
b=a·c
y lo escribiremos como:
a|b
En este caso diremos que b es un múltiplo de a. Si, además b =
̸ 0, diremos que a es
un divisor de b.

La relación a|b en Z − {0} es casi una relación de orden, la única propiedad que falla
es la simétrica. Si a|b y b|a entonces a = ±b. Si nos restringimos a los enteros positivos
Z>0 = N>0 , entonces si es una relación de orden. En general si a es un divisor de b,
también −a es un divisor de b, por lo que vamos a asumir que, cuando hablemos de un
divisor sin especificar más, nos referiremos siempre a un divisor positivo.

Definición 8.4.3. Un número entero p es primo si p ̸= 0, 1, −1 y sus únicos divisores


son 1, −1, p, −p.

Esta definición de número primo es exclusiva de los enteros. En otras estructuras


algebraicas (anillos) se da la siguiente definición, que es equivalente a la dada en Z, pero
no en un anillo arbitrario. La añadimos por completitud

91
8 Teorı́a del Endomorfismo

Definición 8.4.4. En un anillo R, un elemento p se dice primo si:


• p ̸= 0.
• p no tiene inverso para la multiplicación.
• Siempre que p divida a un producto a · b, se tiene que p divide a a o p divide a b.
De nuevo, si p es primo, −p también será primo, por lo que usualmente solo trabajaremos
con primos positivos.
Definición 8.4.5. Sean a, b =
̸ 0, diremos que n es un divisor común de a y b si n|a y
n|b.
Un número d es un máximo común divisor de a y b si:
• Es un divisor común, d|a y d|b.
• Es el mayor para la relación de divisibilidad: Si c es un número tal que c|a y c|b
entonces c|d.
El máximo común divisor es único salvo el signo, por lo que tomaremos siempre el
positivo. Si d es el máximo común divisor de a y b lo denotaremos por d = gcd(a, b) o
d = mcd(a, b).
Por convenio, gcd(a, 0) = gcd(0, a) = a y gcd(0, 0) = 0.
Definición 8.4.6. Sean a, b ̸= 0, diremos que n es un múltiplo común de a y b si a|n
y b|n.
Un número r es un mı́nimo común múltiplo de a y b si:
• Es un múltiplo común, a|n y b|n.
• Es el menor número con esta propiedad para la relación de divisibilidad: Si c es un
número tal que a|c y b|c entonces n|c.
El mı́nimo común múltiplo es único salvo el signo, por lo que tomaremos siempre el
positivo. Si n es el mı́nimo común múltiplo de a y b lo denotaremos por n = lcm(a, b) o
d = mcm(a, b).
Por convenio, lcm(a, 0) = lcm(0, a) = 0 y lcm(0, 0) = 0.
Conocidas factorizaciones en primos de a y b se puede probar que el máximo común
divisor de a y b se obtiene con los primos comunes con menores exponentes y el mı́nimo
común múltiplo con comunes y no comunes con mayores exponentes. No vamos a ver
estos resultados y vamos a ver una manera alternativa de calcular el máximo común
divisor. Esta manera resulta en un cálculo mucho más rápido del máximo común divisor
de dos números enteros. Se basa en el siguiente resultado.
Definición 8.4.7. Sean a, b ∈ Z, b ̸= 0, diremos que q es el cociente y r es el resto de
la división de a entre b si
a = bq + r
0 ≤ r < |b|

92
8.4 Números enteros

Teorema 8.4.8. Sean a, b ∈ Z, b ̸= 0, el cociente y el resto de la división existen y son


únicos.
Demostración. Primeramente, si a ≥ 0 y b > 0 tenemos la división con resto que existe y
es única para números naturales por el Teorema ??.
Para el resto de posibilidades, si
a = bq + r, 0≤r<b
entonces:
a = (−b)(−q) + r, 0 ≤ r < | − b| = b.
El cociente es −q y el resto es r.
−a = b(−q) − r, −b < −r ≤ 0
Si r = 0, el cociente es −q y el resto es 0. Si r < 0
−a = b(−q − 1) + (b − r), 0 < b − r < b.
El cociente es −q − 1 y el resto es b − r. Por último
−a = (−b)q − r, −b < −r ≤ 0.
Si r = 0, el cociente es q y el resto es 0. Si r < 0
−a = (−b)(q + 1) + (b − r), 0 < b − r < b = |−b|.
El cociente es q + 1 y el resto es b − r.
Compárese la siguiente demostración de la unicidad con la dada en ??. La diferencia
está en que en Z podemos restar, lo que simplifica el razonamiento.
Sean c, d ∈ Z, Supongamos que q1 , q2 , r1 , r2 son números enteros tales que:
dq1 + r1 = c, dq2 + r2 = c
0 ≤ r1 < |d|, 0 ≤ r2 < |d|
Queremos probar que q1 = q2 y r1 = r2 .
Podemos suponer, sin pérdida de generalidad que r1 ≤ r2 . Ası́,
dq1 + r1 = dq2 + r2
agrupando términos
d(q1 − q2 ) = r2 − r1 , 0 ≤ r2 − r1 ≤ r2 < |d|
tenemos que r2 − r1 es un múltiplo de |d| =
̸ 0 más pequeño que él. Por tanto debe ser
cero y r2 = r1 . De aquı́
dq1 = dq2
y, como d ̸= 0, podemos cancelar y
q1 = q2

93
8 Teorı́a del Endomorfismo

Ejemplo 8.4.9. El cociente y resto de −13 y 17 son q = −1 y r = 4

−13 = 17(−1) + 4

Observación 8.4.10. La exigencia de que el resto sea mayor o igual a cero es usual en
matemáticas, pero no tiene por que ser la única alternativa, una exposición de alternativas
se puede encontrar en https://en.wikipedia.org/wiki/Modulo_operation (página
consultada en noviembre de 2021).
Vamos a ver cómo la división con resto nos permite calcular el máximo común divisor
de dos enteros sin necesidad de calcular una factorización.

Lema 8.4.11. Sean a = bq + r la división con resto de a entre b, con a y b no nulos.


Entonces
gcd(a, b) = gcd(b, r)

Demostración. Primeramente, si r = 0, entonces b|a y gcd(a, b) = b = gcd(b, 0).


Si r =
̸ 0, veamos que los divisores comunes de a y b son exactamente los divisores
comunes de b y r, por lo que el máximo común divisor coincidirá.
Si d|a y d|b, entonces a = dt1 y b = dt2 , de donde r = a − bq = d(t1 − qt2 ) y d|r, es un
divisor común de b y r.
Recı́procamente, si e|b y e|r, entonces b = es1 y r = es2 , de donde a = bq+r = e(s1 q+s2 ,
por lo que e es un divisor común de a y de b.

Este lema nos permite dar un procedimiento para calcular el máximo común divisor
de a y b conocido como Algoritmo de Euclides.

Algoritmo de Euclides

Input: a, b ∈ Z.
Output: gcd(a, b) ≥ 0.
si a = 0 entonces:
devolver |b|
fin de si
si b = 0 entonces:
devolver |a|
fin de si
mientras b ̸= 0 hacer:
Calcular q, r cociente y resto de dividir a entre b.
a←b
b←r
fin de mientras
devolver a

94
8.4 Números enteros

Ejemplo 8.4.12. Calculemos el gcd(a, b) donde a = 13623, b = −1332. Primeramente,


ni a ni b son cero, por lo que debemos dividir a entre b. Obteniendo:

13623 = (−1332)(−10) + 303

gcd(13623, −1332) = gcd(−1332, 303). Dividimos −1332 entre 303.

−1332 = 303 · (−5) + 183

gcd(−1332, 303) = gcd(303, 183). Dividimos 303 entre 183.

303 = 183 · 1 + 120

gcd(303, 183) = gcd(183, 120). Dividimos 183 entre 120.

183 = 120 · 1 + 63

gcd(183, 120) = gcd(120, 63). Dividimos 120 entre 63.

120 = 63 · 1 + 57

gcd(120, 63) = gcd(63, 57). Dividimos 63 entre 57.

63 = 57 · 1 + 6

gcd(63, 57) = gcd(57, 6). Dividimos 57 entre 6.

57 = 6 · 9 + 3

gcd(57, 6) = gcd(6, 3). Dividimos 6 entre 3.

6=3·2+0

El resto es cero gcd(6, 3) = gcd(3, 0) = 3. Por lo que

gcd(13623, −1332) = 3.

Observación 8.4.13. En el ejemplo anterior, tenemos las factorizaciones:

13623 = 3 · 19 · 239
1332 = 22 · 32 · 37

En el método de Euclides, aunque parezca que se hacen muchas divisiones, es realmente


más eficiente que el conocido método de la factorización. Por ejemplo, en el anterior
método, hemos necesitado hacer 8 divisiones con resto. Si quisiéramos factorizar 13623
por el método de la criba de Eratóstenes, necesitarı́amos dividir por números primos
13623. En total necesitarı́amos 54 divisiones y, para factorizar −1332, necesitarı́amos 16
divisiones, luego 70 divisiones en total.

95
8 Teorı́a del Endomorfismo

Observación 8.4.14. Si solo queremos calcular el máximo común divisor de dos números,
tenemos que gcd(a, b) = gcd(−a, b) = gcd(a, b) = gcd(−a, −b) ¿Puedes probar esto? Por
lo que podrı́amos prescindir de los signos y calcular, en el ejemplo anterior

gcd(13623, 1332) = 3.

Para calcular el mı́nimo común múltiplo, nos basamos en la siguiente propiedad:

Teorema 8.4.15. gcd(a, b) · lcm(a, b) = |a · b|. Luego lcm(a, b) = | gcd(a,b)


ab
|.

Ası́, en el Ejemplo 8.4.12, lcm(13623, −1332) = 13623·1332


3 = 6048612.

Observación 8.4.16. No presenta ninguna dificultad calcular el máximo común divisor


y mı́nimo común múltiplo de una cantidad finita de elementos. Para ello usamos la
propiedad:

gcd(a1 , . . . , an−1 , an ) = gcd(a1 , . . . , an−2 , gcd(an−1 , an ))

lcm(a1 , . . . , an−1 , an ) = lcm(a1 , . . . , an−2 , lcm(an−1 , an ))

Ası́,

gcd(2, 12, 16) = gcd(2, gcd(12, 16)) = gcd(2, 4) = 2.

lcm(15, 5, 7) = lcm(15, lcm(5, 7)) = lcm(15, 35) = 105.

Teorema 8.4.17. Sean a, b ∈ Z. Entonces, existen enteros u, v ∈ R, llamados coefi-


cientes de Bezout, tales que gcd(a, b) = a · u + b · v. Además, para todo r, s, ar + bs es
un múltiplo de gcd(a, b).

El algoritmo para calcular los coeficientes de Bezout de dos elementos es el Algoritmo


extendido de Euclides.

96
8.4 Números enteros

Algoritmo extendido de Euclides

Input: a, b ∈ N>0 .
Output: d, u, v, donde d = gcd(a, b) ≥ 0 y d = a · u + b · v.
si a = 0 entonces:
devolver b, 0, 1
fin de si
si b = 0 entonces:
devolver a, 1, 0
fin de si
F1 ← r0 = a · (1) + b · (0)
F2 ← r1 = a · (0) + b · (1)
i←1
mientras ri ̸= 0 hacer:
Calcular qi+1 , ri+1 cociente y resto de dividir ri−1 entre ri .
Fi+1 ← Fi−1 − qi+1 Fi

// Fi−1 : ri−1 = a·ui−1 + b·vi−1


// Fi : ri = a·ui + b·vi
// Fi−1 − qi+1 Fi : ri+1 = a·(ui−1 − qi+1 ui ) + b·(vi−1 − qi+1 vi )
i←i+1
fin de mientras
devolver ri−1 , ui−1 , vi−1

Ejemplo 8.4.18. Vamos a calcular el máximo común divisor y los coeficientes de Bezout
de 1054 y 85. Primeramente:
1054 = 1054·(1) + 85·(0)
85 = 1054·(0) + 85·(1)
Ahora 1054 = 12 · 85 + 34. Si a la primera ecuación le restamos 12 veces la segunda
obtenemos:
1054 = 1054·(1) + 85·(0)
85 = 1054·(0) + 85·(1)
34 = 1054·(1) + 85·(-12)
La siguiente división es 85 = 34 · 2 + 17. A la fila del 85 le resto dos veces la fila del 34:
1054 = 1054·(1) + 85·(0)
85 = 1054·(0) + 85·(1)
34 = 1054·(1) + 85·(-12)
17 = 1054·(-2) + 85·(25)

97
8 Teorı́a del Endomorfismo

Finalmente 34 = 17 · 2 + 0. El último resto no nulo es 17. Tenemos que:

gcd(1084, 85) = 17 = 1085 · (−2) + 85 · (25).

Aunque no sea parte del temario, adelantar que este mismo algoritmo se puede emplear
para calcular el máximo común divisor de dos polinomios en una variable con coeficientes
en un cuerpo.

Ejemplo 8.4.19. Usemos el mismo algoritmo para calcular el máximo común divisor de
dos polinomios: Sean f (x) = x4 +x3 +5x2 +2x+6, g(x) = x4 +2x3 +4x2 +3x. Escribimos:
f = f · (1) + g · (0)
g = f · (0) + g · (1)
Si dividimos f = gq2 + r2 , con q2 = 1, r2 = −x3 + x2 − x + 6. Si a la primera fila le
restamos la segunda (multiplicada por 1), tenemos:
g = f · (0) + g · (1)
r2 = f · (1) + g · (−1)
La siguiente división g entre r2 tenemos que g = r2 q3 + r3 con q3 = −x − 3,
r3 = 6x2 + 6x + 18. Si a la fila de g le restamos la fila de r2 multiplicada por (−x − 3)
tenemos:
g = f · (0) + g · (1)
r2 = f · (1) + g · (−1)
r3 = f · (x + 3) + g · (−x − 2)
En este caso el resto de dividir r2 entre r3 es cero, luego el máximo común divisor es r3
y los coeficientes de Bezout son x + 3 y −x − 2. Si queremos el máximo común divisor
mónico dividimos toda la expresión por 6.
1 1 1 1
   
x2 + x + 3 = x+ ·f + − x− ·g
6 2 6 3

98

También podría gustarte